Você está na página 1de 133

Chapter 1-1 Organic Chemistry

What is it? Why is it a two semester course? Why is it important to careers in health care?

What is Organic Chemistry?

Chem 61

Organic chemistry is essential to the understanding of the intricate details of life The interactions and reactions of organic molecules are what define living systems. One needs to understand bonding, structure, properties, reactions, and synthesis to understand natural systems. CHAPTER 1 INTRODUCTION Organic Chemistry: the study of carbon compounds... organic compounds contain the elements C, H, N, O, S, Cl, Br, etc. Organic compounds were originally thought to come only from living organisms...thus the term organic (until about 1830) Inorganic compounds were all those which came from non-living sources Scientists thought a vital force was necessary to produce organic compounds and that they could not be synthesized in the lab. In 1828 Fredreich Woeller synthesized urea, an organic compound excreted as waste, from ammonium cyanate, and vitalism slowly died out. Organic compounds range from methane , CH4 (natural gas) to DNA, the genetic coding material, and taxol, a plant derived substance which is a potential anticancer agent. Organic chemistry is fundamental to many scientific disciplines...biochemistry, polymer chemistry, microbiology, botany, pharmacy, medicine...since living systems are composed primarily of organic compounds and water.

Chapter 1-2 STRUCTURAL THEORY AND BONDING

Electronic Structure of Carbon

Chem 61

Carbon is intermediate in electronegativity, therefore it neither completely donates or completely accepts electrons. As a result it forms covalent bonds to itself and other atoms. It can bond to itself to form chains and rings...catenation. This allows the formation of a staggering number of organic compounds. 95% of all known compounds are organic.

ATOMIC STRUCTURE
It is important to understand molecular structure to understand reactivity of organic compounds. Molecular structure depends on atomic structure. Atomic structure of carbon C: 1s22s22p2 that is, carbon has 2 electrons in the lowest energy level, the 1s orbital 2 electrons in the next energy level, the 2s orbital 2 electrons in the third energy level, the 2p orbital 1s, 2s, 2p orbitals 1s is spherical with the same phase throughout

1s 2s is spherical with a node node is where 2 = 0 2s 2p... three orbitals of equal energy (degenerate)

node

2px

2py

2pz

Chapter 1-3

Rules for Electronic Configuration

Chem 61

Pauli Exclusion Principle: Maximum of two electrons per orbital. Electrons have a spin of +1/2 or -1/2 which gives rise to a small magnetic field since electrons are charged. Repulsion between the electrons is reduced if they have opposite spins and thus opposite magnetic moments. If two electrons occupy the same orbital they must have opposite spins or be paired. Aufbau Principle: orbitals are filled from lower to higher energy 4s 3p 3s 2p 2s 1s __ __ __ __ __ __ __ __ order of filling of atomic orbitals __ __

THUS: Li 1s22s1 Hund's Rule: orbitals of equal energy (degenerate) receive one electron until there is one electron in each orbital: then pairing of electrons begins. THUS: C 1s22s22px12py1

Chapter 1-4
ATOMIC RADIUS

Atomic Radius and Electronegativity

Chem 61

Atomic Radius: distance between the nucleus and the outermost electrons (valence electrons) or one half the bondlength of a diatomic molecule HH bond length is 0.74, atomic radius is 0.37 Atomic radius increases with increasing number of electron shells within an atom and decreases with the increase in the number of protons within an atom Thus atomic radius decreases from left to right within the same row of the periodic table (increasing number of protons in the nucleus) and increases from top to bottom within a group of the periodic table (increasing number of electronic shells) ELECTRONEGATIVITY Electronegativity: a measure of an atom's attraction for outer bonding electrons Electronegativity increases with increasing charge on the nucleus and with decreasing distance between the nucleus and the electrons Thus electronegativity increases from left to right within a row of the periodic table and from bottom to top within a group in the periodic table

Chapter 1- 5
CHEMICAL BONDING

Chemical Bonding

Chem 61

G.N. Lewis put forth the first explanation of the nature of the chemical bond. Ionic Bond..one or more electrons is completely transferred from one atom to another creating ions: a cation (positively charged) and an anion (negatively charged). the ions are held together by electrostatic attractions ionic bonds generally occur between atoms of highly different electronegativies. e.g. Nao e Clo + e Nao + Clo Na+ + e Cl Na+Cl

Covalent Bond...one or more electrons is shared by atoms...atomic orbitals merge into shared or molecular orbitals covalent bonds are usually formed between atoms of similar electronegativities since carbon is intermediate in electronegativity it usually forms covalent bonds Generally...atoms not having the noble gas configuration tend to form bonds such that each atom may obtain the stable noble gas electronic configuration [OCTET RULE] H H C H H single bond H C::O H double bond

4H + :C:

H:C:::C:H triple bond

Atoms of the elements of organic compounds can form a fixed number of bonds. C

.. O ..

..

Chapter 1- 6
COVALENT BONDS

Chemical Bonding

Chem 61

Bond lengths: the distance between two covalently bonded nuclei Bond angle: the angle formed between two covalent bonds Bond dissociation energy: the energy required for homolytic cleavage of a bond; Cl Cl H 3C H Cl + Cl

H 3C + H

two radicals

Heterolytic cleavage: cleavage of a bond to give a cation and an anion; one atom which is part of the covalent bond retains both electrons from the bond Cl+ + Cl Cl Cl H 3C H H 3C + cation + H anion

H (change in enthalpy) for homolytic cleavage of many covalent bonds has been determined H3CH HH BrBr HOH 104 kcal/mole 104 kcal/mole 46 kcal/mole 110 kcal/mole

CH3CH3 CH2=CH2 HCCH

88 kcal/mole 163 ckal/mole 230 kcal/mole

Chapter 1-7
POLAR COVALENT BONDS

Polar Covalent Bonds

Chem 61

covalent bonds between atoms of similar electronegativities are said to be nonpolar bonds since each atom shares the electrons of the bond approximately equally HH CH3CH3 if atoms of different electronegativities form a covalent bond, the more electronegative atom will have a stronger attraction for the electrons and polarize the bond giving a polar covalent bond in which one atom is partially negatively ()charged and one atom is partially positively charged (+)

+ H3CCl CH3NH2

+ HCl

+ H2C=O

+ H3COH

If any entire molecule has an overall dipole, then the molecule is said to be polar. CCl4...nonpolar, polar covalent bonds cancel each other because of tetrahedral symmetry HCCl3...polar H2O, NH3...very polar _ Cl Cl _ Cl _ + dipoles add
_

Cl Cl

Cl C Cl

_ H + H O H +

_ +

dipoles cancel

dipoles add

Chapter 1-8
ATTRACTIONS BETWEEN MOLECULES

Polar Covalent Bonds

Chem 61

ion-ion forces: the attraction between oppositely charged ions and repulsion between like charges very strong...not often encountered in organic compounds van der Waals forces: all dipole-dipole forces are both repulsive and attractive... the distance at which the repulsive forces are minimized and the attractive forces are maximized is called the van der Waals radius Induced dipole-dipole interactions or London forces: small temporary dipoles occur and induce dipoles in another molecule due to small uneven distribution of electron density. Dipole-dipole interactions: permanent dipoles in molecules attract or repel Hydrogen bonding: a specific type of dipole-dipole interaction; very strong occur only between a hydrogen atom bonded to an electronegative atom O, N, S and a lone pair on O, N, S

O H N H

:N :O

7 kcal /mole 2 kcal/mole

N H O H

:N :O

3 kcal/mole 5 kcal/mole

Intermolecular hydrogen bonds increase the boiling points of compounds and increase their solubility in water. Hydrogen bonds also can influence the shapes of biomolecules by internal hydrogen bonding as well as hydrogen bonding between molecules.

Chapter 1-9
CHEMICAL FORMULAS

Formulas and Formal Charge

Chem 61

empirical formula: gives the types and ratios of atoms in a molecule molecular formula: gives the type and actual number of atoms structural formula: gives the type, number and attachment of atoms...the actual structure for example: hexane: C6H14 CH3CH2CH2CH2CH2CH3 molecular formula LEWIS STRUCTURES 1. draw the molecular skeleton 2. count the number of available valence electrons (be sure to account for any overall charge on the species) 3. draw covalent bonds between all the atoms giving as many as possible an octet (duet for hydrogen) 4. assign charges in the molecule FORMAL CHARGE formal charge on an atom = # valence electrons - # shared pairs of 3 electrons - # unshared electrons :O: .. .. 1 .. N:O:H HNO3 Lewis structure: .. .. :O: .. 2 N...5-4-0 = +1 O1...6-2-4 = 0 O2...6-1-6 = -1 .. O3...6-2-4 = 0 .. .. :O:.. H...1-1 = O H2SO4 Lewis Structure -2 SO4 :O:S:O: .. .. .. :O: .. O...6-1-6 = -1 S...6-4-0 = +2 overall charge (+2) + 4(-1) = -2 C3H7 structural formula

empirical formula

Chapter 1-10
Structural Formulas

Formulas and Formal Charge

Chem 61

Lewis Structures: as described above using dots for electrons Line-bond formulas: a line is used to represent two electrons forming a bond (a shared pair) Condensed formulas: bonds are not always shown and atoms of the same type bonded to another atom are grouped together H

C O H line-bond

.. ..

H.. . C::O .. . H Lewis

.. ..

H2C=O

.. ..

condensed HCCH condensed CH2=CH2 condensed

H C C H line-bond H H C C H H line-bond

H:C:::C:H Lewis ..H H.. .C::C.. . H H Lewis

Polygon formulas: polygon formulas are often used to represent cyclic compounds for simplicity; each carbon is assumed to have enough additional hydrogens to give each carbon four bonds H H H H C C H H C H C C H C HH H H

cyclohexane

benzene

cyclopropane

cyclobutane

cyclopentane

decalin

Chapter 1-11
ACIDS AND BASES Bronsted-Lowry: acid: a proton donor base: a proton acceptor Strong acid: completely ionized or dissociated in water e.g., HCl, H2SO4, HNO3, HBr Weak acid: only partially dissociated in water: carboxylic acids are weak acid CH3CO2H acid + H2O base CH3CO2conjugate base + H3O+ conjugate acid

Acids and Bases

Chem 61

amines are weak bases CH3NH2 base + H2O acid CH3NH3+ + conjugate acid HO conjugate base

Generally: strong acids have weak conjugate bases and weak acids have strong conjugate bases that is, as acid strength increases, the basicity of the conjugate base decreases Thus the ability of the conjugate base to stabilize a negative charge determines the strength of an acid Conjugate Acids H 2O pKa 15.75 HCN 6.37 CH3CO2H 4.75 H3PO4 2.12 HCl -7

increasing acid strength Conjugate Bases HO NC CH3CO2 H2PO4 C l

decreasing base strength

Chapter 1-12
Factors affecting Acidity:

Acids and Bases

Chem 61

the electronegativity and the size of the atom which carries the negative charge influence its ability to stabilize the negative charge size of the atom HF pKa 3.45 HCl -7 HBr -9 HI -9.5

increasing size of halogen, increasing acid strength electronegativity pKa (CH3)3CH 50 (CH3)2NH 35 CH3OH 15.5 FH 3.45

increasing electronegativity of atom, increasing acid strength ACIDITY CONSTANTS, Ka's for acetic acid CH3CO2H + H2O [CH3CO2] [H+] [CH3CO2H] CH3CO2 + H3O+

Ka =

since stronger acids are more ionized, the larger Ka, the stronger the acid pKa = -logKa, the lower the pKa, the stronger the acid also, the higher the pKa, the weaker the acid or the stronger the base the stronger the acid, the more stable the anion produced by ionization of the acid.

Chapter 1-13
LEWIS ACIDS AND BASES

Lewis Acids and Bases

Chem 61

Lewis acid: electron pair acceptor: any species with an electron deficient atom BBr3, AlCl3, H3C+

Lewis base: electron pair donor; any species with an unshared pair of electrons .. .. H3N:, CH3CH2.. OH, H2C=O:

Chapter 1-14
QUANTUM MECHANICS...Molecular Orbitals

Quantum Mechanics

Chem 61

In 1923 Louis De Broglie postulated that electrons have properties of three dimensional waves Later a wave equation was developed. Solutions () to this wave equation give the various electronic states known as atomic orbitals. = probability of finding an electron in a certain space = electron 2 probability density plots of give the familiar s,p,d...orbitals
2

WAVE PROPERTIES The amplitude of a wave may be above the resting state (positive) or below (negative)...no charge implied A node is a point at which the amplitude is zero Waves reinforce creating a wave of higher amplitude if they they are in phase.

+ + +

Waves interfere if they are out of phase and create a wave which is of lower amplitude. Complete interference results in the cancelling of one wave by another.

Chapter 1-15
1s, 2s, 2p orbitals 1s is spherical with the same phase throughout +

Quantum Mechanics

Chem 61

2s is spherical with a node node is where 2 = 0 + node

2p... three orbitals of equal energy (degenerate) node

2px

2py

2pz

4s __ 3p __ __ __ 3s __ 2p __ __ __ order of filling of atomic orbitals 2s __ 1s __

Chapter 1-16
MOLECULAR ORBITALS Molecular orbitals = Linear combination of atomic orbitals

Molecular Orbitals

Chem 61

2 Atomic Orbitals must produce 2 Molecular Orbitals (the number of molecular orbitals equals the number of atomic orbitals which were combined to form them) the hydrogen molecule * H1s E 1 bonding 2 1 + 2 antibonding 1 2

>E H1s H2

. ..

the energy of the hydrogen molecule with two electrons in the sigma orbital is 104 kcal/mole more stable than the separate hydrogen atoms; E = 52 kcal/mole if one electron is in the sigma and one in the sigma*, the molecule is of higher energy than the two separate atoms because the s* is slightly >E higher than the s orbitals while the s is E lower Bonding orbital...high electron density between nuclei Antibonding orbital..node between nuclei (zero electron density) bond is cylindrically symmetrical Aufbau principle...fill lowest energy orbitals first Hund's Rule...place one electron in each degenerate orbital first, then pair up Pauli Exclusion Principle...two electrons in the same orbital must have opposite spins

Chapter 1-17
MOLECULAR ORBITALS ON CARBON

Molecular Orbitals of Carbon

Chem 61

Overlap between atomic orbitals in complex molecules often results in electron repulsions which destabilize the molecule. Hybrid orbitals allow for better overlap and a more accurate prediction of molecular structure. Methane CH4 has a central carbon with four equivalent bonds to hydrogen bond angles = 109.5 bond lengths = 1.09 sp3 Hybridization carbon has electronic configuration 1s22s22p2

2p 2s 1s + 2s + + 3-3p's 96 kcal

2p 2s 1s hybridize sp3 1s

109.5 apart 4-sp3's

sp3 orbitals point toward the corners of a tetrahedron, 109.5 apart any carbon bonded to four other atoms is sp3 hybridized, e.g. CH4, H3CCH3, CC bond length = 1.54 C-H bonds require 104 kcal/mol to be broken methane H C H H H

bond angles = 109.5; tetrahedral

Chapter 1-18
sp2 Hybridization

sp2 Molecular Orbitals of Carbon

Chem 61

sp2 hybridized carbons are trigonal planar with atoms 120 apart 2p hybridize 1s sp2

2p 2s 1s 96 kcal

2p 2s 1s

ethylene: trigonal planar H H H H H H 2p H C H pi orbital C H H sp2 H C H pi* orbital bond angles = 120; C=C bond length = 1.34 pi () bonds are formed by the side by side overlap of two p-orbitals (approx. 68 kcal/mol) pi bonds are above and below the plane where the sigma bond is located pi bonds make the molecule rigid between the two atoms preventing rotation sigma * pi * pi sigma ethylene C=C double bond: one sigma, one pi C H H H C C H H sigma* orbital H H H C C H

C 120

H sigma orbitalH

Chapter 1-19
sp Hybridization sp hybridized carbons are linear with atoms 180 apart

sp2 Molecular Orbitals of Carbon

Chem 61

2p 2p 2s 1s 96 kcal 2p 2s 1s hybridize 1s sp

ethylene: trigonal planar + 2s 180 apart; the remaining two 2p orbitals are 90 to the sp and each other

+ 1-2p 2-sp's

Acetylene: linear; bond angles 180 C=C bond length = 1.20 acetylene has two perpendicular pi bonds and one sigma bond

H 2-sp's

acetylene -orbital

2-2p's on each carbon combine to form pi-orbitals CC * CC * CC CC

acetylene -orbitals

Chapter 1-20
FUNCTIONAL GROUPs CH3CH3 alkanes CH3 OH alcohols CH2 CH2 alkenes CH3 OCH3 ethers O CH3 C CH3 ketones HC CH alkynes CH3 NH2 amines

Functional Groups
Oxygen: sp3 2p 2s CH3 Br alkyl halides 1s O: 1s22s22p4 hybridize 1s sp3

Chem 61

O CH3 C H aldehydes

O CH3 C OH carboxylic acids

O CH3 C OCH3 esters .. .. O H H .. .. O H CH2CH3 ethanol

two sp3 orbitals already filled with an unshared pair of electrons bonding can occur to two other atoms sp3

.. .. O CH CH 2 3 CH2CH3 diethyl ether ..

.. O

sp2 CH3 C CH3

Hybrid orbitals of oxygen and nitrogen Nitrogen: sp3

water

acetone

2p 2s 1s N: 1s22s22p3 hybridize 1s

sp3

one sp3 orbital already filled with an unshared pair of electrons bonding can occur to three other atoms .. N H H H .. CH3 H methyl amine N

H ammonia

Chapter 1-21
RESONANCE STRUCTURES Chapter 6; Bruice, Pages 260 - 282:

Resonance Structures

Chem 61

Some molecules cannot be accurately represented by one simple line-bond formula: they are "hybrids" of two or more structures

O O C O

O O C O

O O C O

2/3

O 2/3 O C O 2/3

1. Resonance structures exist only on paper...the actual structures are hybrids of all the resonance structures 2. Resonance structures differ only in the position of electron pairs....not atoms 3. All structures should be proper Lewis structures (exceptions) 4. All resonance structures should have the same number of unpaired electrons

Nonequivalent Resonance Structures 1. Structures with a maximum number of octets is preferred. 2. Charges should be located on atoms with compatible electronegativity. 3. Minimize charge separation. 4. Charge separation may be enforced by the octet rule (atoms may be charged if they have an octet.)

Chapter 2-1
CHAPTER 2

Hydrocarbons

Chem 61

Hydrocarbons: compounds containing only hydrogen and carbon: alkanes, alkynes, alkenes alkanes contain only CH and CC single bonds CnH2n+2; alkenes CnH2n contain C C double bonds and alkynes CnH2n-2 contain CC triple bonds Alkanes do not react with hydrogen, but alkenes and alkynes can react with hydrogen under certain conditions H2, catalyst CH2 CH2 alkene 2H2, catalyst CH3 C C CH3 alkyne ISOMERISM Structural Isomers: compounds with the same molecular formula that differ in the order in which the atoms are bonded to one another CH3 C CH3 CH3

CH3 CH3 alkane H3CH2C CH2CH3

CH3

H CH3 CH3 C C CH3 H H C5H12 2-methylbutane

H H H CH3 C C C CH3 H H H C5H12 n-pentane

C5H12 2,2-dimethylpropane

Chapter 2-2
NOMENCLATURE Alkanes are named by the following parent names CH4 CH3CH3 CH3CH2CH3 CH3(CH2)2CH3 CH3(CH2)3CH3 CH3(CH2)4CH3 CH3(CH2)5CH3 CH3(CH2)6CH3 CH3(CH2)7CH3 CH3(CH2)8CH3 methane ethane propane butane pentane hexane heptane octane nonane decane C1 C2 C3 C4 C5 C6 C7 C8 C9 C10

Nomenclature

Chem 61

Cyclic alkanes are named the same but with cyclo as a prefix

cyclohexane

cyclobutane

Branched hydrocarbons are named from the parent with a substituent as a prefix hydrocarbon branches are named by dropping ane from the parent and adding -yl thus methane: methyl ethane: ethyl propane: propyl , etc.

special trivial names for branches H CH3 C C CH3 H H isobutyl H CH3 C C CH3 H H sec-butyl 23

CH3 C CH3 H isopropyl

CH3 C CH3 CH3 tert-butyl

Chapter 2-3
BASIC RULES OF NOMENCLATURE

Nomenclature

Chem 61

1. Find the longest continuous chain (not necessarily drawn as a straight line) and name the parent 2. Number the parent chain starting at the end nearest the branch 3. Identify the branch and its position 4. Attach the number and the name of the branch to the parent name. H CH3 C 3 4 H CH3 the branch is CH3; therefore methane butane is the parent methyl

C CH3 2 1 H

2-methylbutane H CH3 C H 3C H C H CH3H C C CH3 3,3,5-trimethylhexane CH3H

Other Functional Substitutents CO2H C H O C increasing priority O OH NR2 C=C CC R, C6H5, Cl, Br, NO2 -oic acid -al -one -ol -amine -ene -yne prefix substituents

24

Chapter 2-4
alkenes

Nomenclature

Chem 61

H CH3 C

H C C H C CH3 2,5-dimethyl-2-hexene

CH3H

CH3

alcohols

H CH3 C HO

H C H

CH3 C H H 2-pentanol

carboxylic acids

H H C Br

H C H

H C H COOH 4-bromobutanoic acid

aldehydes and ketones

H CH3 C H

H C

CH2CH3 C CHO CH3

H C H

O C

CH2CH2CH3 C H CH3

CH3H

2-ethyl-3-methylpentanal

4-methyl-3-heptanone

25

Chapter 2-5
ALKANES Physical Properties nonpolar compounds C1 to C4 are gases; C5 to C17 are liquids; >C17 are solids Boiling points increase about 30C for each additional CH2 unit

Alkanes

Chem 61

branching lowers the boiling point due to disruption of van der Waals attractions insoluble in water; soluble in organic solvents like diethyl ether, benzene Chemical Properties very unreactive compounds Halogenation CH3CH3 + Cl2 light

CH3CH2Cl + HCl + other products

Oxidation of Alkanes Combustion spark CH3CH2CH2CH2CH3 + 8O2 5CO2 + 6H2O

oxidation: a reaction that either removes a hydrogen atom from a carbon or adds an electronegative element to the molecule (O, N, S, halogen). low oxidation level CH2=CH2 CH3CH2OH CH3CH2NH2 CH3CH2Cl HCCH CH3CH=O CH3CH=NH CH3CHCl2 high oxidation level

CH3CH3

CH3CO2H CH3CN CH3CCl3

CO2 CCl4 26

Chapter 2-6
"Complete"/"incomplete" oxidation (combustion) of propane CH3CH2CH3 2 CH3CH2CH3 CH3CH2CH3 5O2 7O2 2O2

Oxidation and Reduction

Chem 61

3 O=C=O + 4 H2O "complete" combustion 6 CO + 8 H2O 3 C + 4 H2O "incomplete" combustion "incomplete" combustion

Heat of Combustion: energy released when a compound is completely oxidized to CO2 and water; depends mostly on number of CH2 units: approximately 157 kcal/ methylene unit Reduction of Alkenes, Alkynes reduction: a reaction that either adds H atoms or removes an electronegative atom from the molecule Pd, Ni,or Pt; H2 Pd, Ni,or Pt; H2 Pd, Ni,or Pt; H2

CH3CH3 CH2=CH2 HCCH

no reaction (NR) CH3CH3 CH2=CH2 Pd, Ni, Pt H2 CH3CH3

Pd, Ni,or Pt; H2 heat

Chapter 2-7
CONFORMATIONS OF OPEN CHAIN COMPOUNDS Molecular Mechanics

Conformations of Acyclic Hydrocarbons


Butane

Chem 61

E eclipsed

Steric Energy: (isolated molecule in gas phase at 0 K): relative energy of a conformation or stereoisomer calculated using classical mechanics (atoms and bonds treated as balls and springs) Stretch (bond length): energy associated with stretching or compressing bonds from their optimal length Bend (bond angle): energy associated with deforming bond angles from their optimal angle Stretch-Bend: energy required to stretch two bonds involved in a severely compressed bond angle dipole-dipole: energy associated with interaction of bond dipoles out of plane: energy required to distort a trigonal center out of planarity torsional strain: destabilization from eclipsing of bonds on adjacent atoms van der waals strain: destabilization from two atoms being too close together dimensional Ethane H H C H H C H staggered H H H H H staggered H H HH eclipsed HH H H Newman projection H H CH3 H H H H CH3 anti H CH3 H HCH3 eclipsed1 H

eclipsed methyls
4.5 kcal 3.8 kcal

gauche 0

0.9 kcal

anti

60

120

180

240

300 360

H H CH3 CH3 gauche H H H H H CH3

H HCH3

H H

H3CCH3 eclipsed2

H CH3 gauche

HCH3 eclipsed3

E eclipsed

Ethane
staggered 0 60 120 180 240

H = 3 kcal/mole
0 kcal

300 360

Chapter 2-8
CYCLIC COMPOUNDS Strain energy

Conformations of Cyclic Hydrocarbons

Chem 61

-H (kcal/mole) cyclopropane cyclobutane cyclopentane cyclohexane 499.8 655.9 793.5 944.5

-H per CH2 166.6 164.0 158.7 157.4

strain energy per CH2 9.2 6.6 1.3 0

total strain energy 27.6 26.4 6.5 0

cyclopropane: bond angles 60; tetrahedral 109.5

sp3 orbitals are 109.5 apart cyclopropane bond angles 60 maximum overlap cannot be achieved

cyclobutane and cyclopentane H H H H H H puckered cyclobutane H H H H H H H H HH H H

envelope cyclopentane

puckering allows bond angles to be at or close to the tetrahedral angle and minimizes torsional strain (electronelectron repulsions in eclipsed bonds) between adjacent CH bonds

Chapter 2-9
cyclohexane H H H H H chair cyclohexane H H H H H H equatorial bonds H

Conformations of Cyclohexane

Chem 61

axial bonds half chair boat

E twist boat chair 7.1 5.5 10.8 kcal

60

120

180

240

300

360

H H H H H H H H H

H H H

H H H H H H H H

H H H

H H

H H H

chair

half chair

H H H H H H H

boat

chair twist boat

Chapter 2-10
Substituted Cyclohexanes

Conformations of Cyclohexane

Chem 61

substitutents on cyclohexanes preferntially occupy equatorial positions due to 1,3 diaxial interactions in axially substituted cyclohexanes H H H H H H

H H H

CH3

more stable by 1.8 kcal/mole

H H 1,3-diaxial interactions

H H

gauche H CH3 C H C H H H

H H H H C C

H H anti CH3 H

axial substitution similar to gauche butane equatorial substitution similar to anti butane

CH3 CH3 H CH3 C H H H H

H H

C(CH3)3 H

5.6 kcal/mole more stable

G = -RTlnK G = -(1.98 kcal/mol)(298)(2.94) = - (1.98 kcal/mol)(298)(ln19) = -1.74 kcal/mole for methylcyclohexane

Chapter 3-1
Chapter 3 STEREOCHEMISTRY Geometric isomerism in cyclic compounds

Stereochemistry: Geometric Isomers

Chem 61

OH H OH OH trans-1,2-cyclohexanediol CH3

OH

OH

H cis

OH

H CH3 trans-1,3-dimethylcyclobutane H Br Br cis Geometric Isomerism in alkenes 68 kcal/mole to cleave a carbon-carbon pi bond thus no "free" rotation H C CH3 C H CH3 CH3 H C C CH3 H

CH3

Br cis

trans-2-butene

cis-2-butene

H C CH3CH2 C

CH3 H

H C Cl C

H Cl

trans-2-pentene

cis-1,2-dichloroethene

Chapter 3-2

Absolute Configuration

Chem 61

Stereoisomers: compounds with the same structures differing only in their arrangement of atoms in space. not cis or trans CH3 C CH3CH2 C H Cl Cl C Br C I F

E entgegen (across)

Z zusammen (together)

Cahn-Ingold-Prelog Sequence rules 1. if the atoms are different, highest atomic number gets highest priority 2. if two isotopes of the same element, the one with the higher mass gets higher priority 3. if the atoms are the same, the atomic numbers of the next atoms are used to assign priority 4. atoms attached by double or triple bonds are given single bond equivalencies

O R C R' =

(O) R C R' O (C) (C) R

O C OH = R

(O) C O (C) OH (C)

CR'

R R'

C (C) C (C) (C)

C H

CR' H

R H

C C (C)

H R'

Chapter 3-3
CHIRALITY

Stereochemistry: Chirality

Chem 61

An object or molecule which cannot be superimposed on its mirror image is said to be chiral If an object or molecule can be superimposed on its mirror image, it is achiral. Enantiomers: isomers which are nonsuperimposable mirror images. mirror plane

0000 0000 0000

Cl

Cl

Br

Br

Stereogenic carbon atom: a carbon with four different groups bonded to it (designated *). H Cl C* CH3 CH3 H C* Cl CH3

OH C* CH2CH3 CH2CH2CH3 H

CH2CH3 CH2CH3 enantiomers Fischer Projections

by convention: horizontal bonds come out of the paper vertical bonds go back into the paper CH3 H C OH CH3 H OH CH2CH3 H H C OH H C OH CH3 H H H OH OH CH3

CH2CH3

0000 0000 0000 0000 0000 0000 0000 0000 000 000 000 0000 000 0000 0000 0000 0000 0000 0000 0000 0000 0000
Cl Cl I Br Br H I H

000 000 000 0000 000 0000 0000 0000

000 000 000 000

000 000 000 000 000 000 000 000

Br C* CH3

Chapter 3-4
OPTICAL ROTATION

Stereochemistry: Chirality

Chem 61

enantiomers have almost all the same physical and chemical properties properties which differ are: 1. interaction with other chiral substances 2. interactions with polarized light

Polarimeter

ordinary light

polarized light

rotated light

lamp

polarizer

solution of sample

if plane polarized light is passed through a solution of a single enantiomer, the light is rotated either to the right or the left; the opposite enantiomer will rotate the light in the opposite direction optically active: a compound which rotates plane polarized light optical isomers: enantiomers dextrorotatory: rotates plane polarized light to the right, also (+) or d levorotatory: rotates plane polarized light to the left, also () or l racemic mixture: a 1:1 (50:50) mixture of two enantiomers does not rotate plane polarized light; therefore optically inactive

Chapter 3-5

Stereochemistry: Absolute Configuration

Chem 61

Absolute configuration: the order of arrangement of the four groups around a stereogenic center enantiomers have opposite configurations (R) and (S) Cahn-Ingold-Prelog System R: rectus or right S: sinister or left To assign R and S to an asymmetric atom 1. Rank the four attached groups from 1 (highest) to 4 (lowest) priority based on the Cahn-Ingold -Prelog Sequence rules 2. Project the molecule with the lowest priority group to the rear 3. Draw a semicircle from 1 to 2 to 3 4. If the direction of the semicircle is clockwise; configuration is R; if counterclockwise; configuration is S 3 CH3 1 Br C H 4 Cl 2 2 CH3CH2 3 CH3 C H R 4 R OH 1

If lowest priority group is out; assign the configuration as usual and then reverse it 2 Cl 1 Br C CH3 3 3 CH3

2 Cl C H R S 4 S Br 1

H 4 lowest priority group out

Chapter 3-6
Molecules with Two or more Asymmetric centers

Stereochemistry: Diastereomers

Chem 61

If a molecule has n asymmetric carbon atoms, it contains a maximum of 2n isomers; may not have that many diastereomers CH2OH H H C C OH OH HO HO CH2OH C C H H H HO diastereomers

meso H CH3 H CH3 H CH3

chiral H CH3

HO H meso

OH H

HO H chiral

H OH

CH2OH C C OH H HO H enantiomers

CH2OH C C H OH

mirror plane

mirror plane

CH3 enantiomers

CH3

CH3

CH3

two asymmetric carbons; 4 isomers diastereomers: stereoisomers which are not enantiomers; may have different physical and chemical properties Meso compounds meso compounds contain an internal plane of symmetry and are achiral: that is the mirror image is identical to the original diastereomers meso: identical CH2OH H H C C OH OH HO HO CH2OH C C H H H HO CH2OH C C OH H HO H CH2OH C C H OH

CH2OH

CH2OH

CH2OH enantiomers

CH2OH

superimposable

Chapter 3-7

Stereochemistry: Asymmetric Synthesis, Resolution

Chem 61

Preparation of Enantiomerically Enriched Compounds Generating chiral compounds from achiral compounds Enzymes O O OEt >98% R HO O OEt

yeast

Asymmetric Reagents H OH t-BuOOH (+)-diethyl tartrate Ti(i-OPr)4 O OH H >95% one enantiomer Resolution of a Racemic Mixture separated salt converted back to acid (R) R*COOH + (S) R*NH2 + (S) R*COOH racemic (R) R*COO(S) R*NH3+ + (S) R*COO(S) R*NH3+ diastereomeric salts (separable) (S) R*COOH

(R) R*COOH

Chapter 4-1
ACIDS AND BASES Bronsted-Lowry: acid: a proton donor base: a proton acceptor Strong acid: completely ionized or dissociated in water e.g., HCl, H2SO4, HNO3, HBr Weak acid: only partially dissociated in water: carboxylic acids are weak acid CH3CO2H acid + H2O base CH3CO2conjugate base + H3O+ conjugate acid

Acids and Bases

Chem 61

amines are weak bases CH3NH2 base + H2O acid CH3NH3+ + conjugate acid HO conjugate base

Generally: strong acids have weak conjugate bases and weak acids have strong conjugate bases that is, as acid strength increases, the basicity of the conjugate base decreases Thus the ability of the conjugate base to stabilize a negative charge determines the strength of an acid Conjugate Acids H 2O pKa 15.75 HCN 6.37 CH3CO2H 4.75 H3PO4 2.12 HCl -7

increasing acid strength Conjugate Bases HO NC CH3CO2 H2PO4 C l

decreasing base strength

Chapter 4-2
Factors affecting Acidity:

Acids and Bases

Chem 61

the electronegativity and the size of the atom which carries the negative charge influence its ability to stabilize the negative charge size of the atom HF pKa 3.45 HCl -7 HBr -9 HI -9.5

increasing size of halogen, increasing acid strength electronegativity pKa (CH3)3CH 50 (CH3)2NH 35 CH3OH 15.5 FH 3.45

increasing electronegativity of atom, increasing acid strength ACIDITY CONSTANTS, Ka's for acetic acid CH3CO2H + H2O [CH3CO2] [H+] [CH3CO2H] CH3CO2 + H3O+

Ka =

since stronger acids are more ionized, the larger Ka, the stronger the acid pKa = -logKa, the lower the pKa, the stronger the acid also, the higher the pKa, the weaker the acid or the stronger the base the stronger the acid, the more stable the anion produced by ionization of the acid.

Chapter 4-3
LEWIS ACIDS AND BASES

Lewis Acids and Bases

Chem 61

Lewis acid: electron pair acceptor: any species with an electron deficient atom BBr3, AlCl3, H3C+

Lewis base: electron pair donor; any species with an unshared pair of electrons .. .. H3N:, CH3CH2.. OH, H2C=O:

Chapter 5-1
ALKENES Alkene Structure sp2 hybridized carbons are trigonal planar with atoms 120 apart 2p 2s 1s 96 kcal 2p 2s 1s hybridize

Alkenes: Structure and Isomerism

Chem 61

2p sp2 1s

ethylene: trigonal planar H H H H H H 2p H C H pi orbital C H H sp2 H C H pi* orbital


bond angles = 120; C=C bond length = 1.34 pi () bonds are formed by the side by side overlap of two p-orbitals (approx. 68 kcal/mol) pi bonds are above and below the plane where the sigma bond is located pi bonds make the molecule rigid between the two atoms preventing rotation

H H

H C C

C 120

H sigma orbitalH

H C

H C C H

H H sigma* orbital

sigma * pi * pi sigma ethylene C=C double bond: one sigma, one pi

Chapter 5-2
Geometric Isomerism in alkenes

Alkenes: Structure and Isomerism

Chem 61

68 kcal/mole to cleave a carbon-carbon pi bond thus no "free" rotation H C CH3 C H CH3 CH3 H C C CH3 H

trans-2-butene

cis-2-butene

H C CH3CH2 C

CH3 H

H C Cl C

H Cl

trans-2-pentene

cis-1,2-dichloroethene

Stereoisomers: compounds with the same structures differing only in their arrangement of atoms in space. not cis or trans CH3 C CH3CH2 C H Cl Cl C Br C I F

E entgegen (across) Cahn-Ingold-Prelog Sequence rules

Z zusammen (together)

1. if the atoms are different, highest atomic number gets highest priority 2. if two isotopes of the same element, the one with the higher mass gets higher priority 3. if the atoms are the same, the atomic numbers of the next atoms are used to assign priority 4. atoms attached by double or triple bonds are given single bond equivalencies

Chapter 5-3
Alkenes: Reactivity

Alkenes: Electrophilic Addition Reactions

Chem 61

The reactivity of alkenes is due to their ability to donate a pair of electrons: their Lewis basicity. H Consider Ethylene: The site of reactivity is the pi-bond due to the exposed nature of the pi electrons H H H H H H 2p H C H pi orbital C H H sp
2

H H C H C H

Br H H 2p

H H H

Br

Br H H

H H H

H H

H C C

H Br + CH2CH2H HBr Br H H2C CH2

C 120

H sigma orbitalH

CH2=CH2

H C H pi* orbital C

H H

H C H C

H H sigma* orbital

The pi electrons act as a Lewis base (electron pair donor). These electrons react with electron deficient species (Lewis acids) Alkenes undergo electrophilic additions reactions. Electrophile: an electron deficient ion or molecule H+ Br+ BH3 +CH3

Nucleophile: an electron rich ion or molecule. I CH3NH2 H2O

Reaction Mechanisms of Electrophilic Addition Reactions reaction mechanism: a detailed description of how a chemical reaction occurs. A roadmap of a reaction....curved arrows show which bonds are formed or broken. A mechanism includes the transition states involved in making and breaking bonds and reactive intermediates that are formed along the pathway from reactants to products.

Chapter 6-1
Thermodynamics describes the properties of a system at equilibrium G = RT ln Keq where Keq = [reactants] [products] R = 1.986 X 10-3 kcal K-1 mol-1 (gas costant) T = temperature in degrees Kelvin Go = Ho TSo free energy = enthalpy T x entropy enthalpy = heat of reaction entropy = state of disorder transition state

Thermodynamics

Chem 61

Energy Diagram

E reactants

products

progress of reaction

reactants

G products

exergonic

+ G reactants

products

endergonic

Chapter 6-2
Kinetics describes the rate of progression of a reaction

Kinetics

Chem 61

depends on the energy of activation (the stability of the transition state): the lower the transition state energy, the faster the reaction will be. transition state slower E reactants progress of reaction first order reaction rate = k[A] rate is proportional to the concentration of one reactant G E products reactants progress of reaction G

faster

transition state

products

second order reaction rate = k[A] [B] rate is proportional to the concentration of two reactants

A two step reaction

transition state E G reactants Rate limiting step: is the slowest step (step with the highest energy of activation) transition state G intermediate products

Chapter 6-3
Electrophilic Addition Reactions

Electrophilic Addition: Addition of HX

Chem 61

Nucleophilic addition does not occur with alkenes unless an electron-attracting group is attached to one of the carbon atoms to cause a polarity difference.

Nu:

No Reaction

However, in the electrophilic addition reaction, the reagent is E+A (E+ = electrophile, A = some anion)

C C

E A

E + C C

A A

C C

Note the alkene acts as a Lewis base (or nucleophile) toward the Lewis acid (or electrophile) The intermediate, a carbocation, reacts with A to yield a product in which E and A have added to the C=C double bond. Addition of HX (E+ = H A = F, Cl, Br, I) X CH3 CH3 X C CH3

CH3

CH2 + HX

CH3

+ C

CH3

CH3

CH3

Reactivity: HI > HBr > HCl > HF The reaction is said to be regioselective since an unsymmetrical alkene gives a predominance of one of two possible electrophilic addition products. The term regiospecific is used if one product is formed exclusively. In these reactions, the halogen (A) is found attached to the most substituted carbon atom of the alkene (Markovnikoff's Rule):

Chapter 6-4
CH3 C CH2 + HX + C X CH3

Electrophilic Addition: Addition of HX


X CH3 C CH3

Chem 61

CH3

CH3

CH3

CH3

C R

CHR + HX

+ C

X CHR R

X C R CHR

C R

CHR + HX

R 3 carbocation more favored X + R CH CR R 2 carbocation less favored

H R C R

X CHR

Thus regioselectivity is explained by the lower Eact leading to the 3 carbocation intermediate. If a nucleophilic solvent is employed in the electrophilic addition reaction solvent may compete with A for the intermediate carbocation. Cl CH CH2 HCl CH3COOH (solvent) CH + CH3 OCOCH3 CH CH3

Note the C=C's of the aromatic ring do not undergo this type of reaction. Since a carbocation intermediate is involved, rearrangement may sometimes occur. + HCl (CH3)2CH CH CH3 (CH3)2CH CH CH2

(CH3)2CH

CH CH3

(CH3)2C Cl

CH2 CH3 60%

40% Cl

Chapter 6-5

Carbocation Stability

Chem 61

Carbocation stability Carbocations are highly reactive intermediates ; they cannot be observed directly in the reaction mixture since they react as soon as they are formed. Any structural feature that will disperse a positive charge will stabilize the carbocation. This stabilization has been quantitated by so-called hydride H: affinity measurements (gas phase). The lower the value of the hydride affinity, the more stable the carbocation. Name methyl primary 1 secondary 2 tertiary 3 vinyl allyl secondary allyl tertiary allyl phenyl + benzyl CH2+ secondary benzyl tertiary benzyl diphenylmethyl triphenylmethyl + CHCH3 + C(CH3)2 + CH + C 226 220 215 210 233 Structure CH3+ CH3CH2+ (CH3)2CH+ (CH3)3C+ CH2=CH+ CH2=CHCH2+ CH2=CHCH+(CH3) CH2=CHC+(CH3)2 Hydride Affinity (kcal/mol) 314 274 247 230 287 256 237 225 298

Chapter 6-6

Carbocation Stability

Chem 61

Thus the order of carbocation stability is triphenyl methyl > diphenylmethyl > 3 benzyl allyl > 2 > 1 >> methyl Carbocation centers can be stabilized by overlap with adjacent pi orbitals (resonance) or adjacent sigma orbitals (hyperconjugation).

H + CH3 CH3 p C .. C H H

sigma donation adjacent sigma bond donates some electron density to the empty p orbital on the carbocation center more adjacent sigma orbitals result in a more stable carbocation thus 3 > 2 > 1 > methyl

+ H H p + CH2 pi C C

H C H H empty p orbital ovelaps with adjacent pi orbital to disperse the positive charge

CH

CH2

CH2

CH

+ CH2

equivalent resonance structures Since the pi orbital is closer in energy to the empty p than the sigma orbital, the pi overlap stabilizes the cation more efficiently.

Chapter 6-7
Addition of Water (E+ = H+; A = HOH) use H2SO4, H2O H+ Step 1 R 2C CHR

Electrophilic Addition: Addition of H2O

Chem 61

+ R 2C

CH2 R H + :O H

..
Step 2 + R 2C H Step 3 H O:
+

:OH2 CH2 R

R 2C

CH2 R

..
:OH2 CH2 R TS1

..
:O H R 2C CH2 R + H3O+

R 2C

TS2 Eact + R 2C CH2 R H +O H R 2C CHR R 2C CH2 R O H R 2C CH2 R TS3

Addition of water to alkenes is a reversible reaction and whether the alkene or the alcohol predominates at equilibriuim depends on the reaction conditions. Low temperatures and high concentrations of water favor the alcohol. Higher temperatures and removal of water favor the alkene.

Chapter 6-8
Addition of Mercury acetate and Water ( E+ = +HgOCOCH3; A = OH2)

Electrophilic Addition: Oxymercuration

Chem 61

If alcohol is used as the solvent in this process ethers are obtained as the products. CH3 CH3 RO

To avoid carbocation rearrangements under the conditions used in addition of H2O (H2SO4/H2O) a better system for the addition of water to a C=C has been devised. O CH3 C O Hg O O C CH3
+

CH3 C CH3 CH2

1. Hg(OCOCH3)2, ROH 2. NaBH4

CH3

O Hg O E
+

O CH3 +

CH3

mercury acetate

CH3 C CH3 CH3 CH3 CH2 + +Hg O

O C CH3 + CH3 C

O CH2 Hg O C CH3 O C CH3

CH3 carbocation or O H2O: CH3 C


+

Hg O CH2

CH2 Hg O C CH3

+OH2

CH3 bridged carbocation

Since no rearrangement is observed in reactions with +HgOCOCH3 the bridged carbocation is believed to be the true intermediate. Attack by H2O on the more electron deficient carbon opens the bridged cation (backside attack). NaBH4 reduction of the CHg bond yields the alcohol in a separate step. CH3 CH3 C CH2HgOCOCH3 +OH2 H 2O CH3 CH3 HO C CH2 HgOCOCH3 NaBH4

CH3 CH3 HO

CH3 + Hg

Chapter 6-9
Hydroboration (E+ = B, A = H)

Electrophilic Addition: Hydroboration

Chem 61

Discovered by H.C. Brown in the 1950's. The reagent BH3 (borane) is used in electrophilic addition reactions. Borane is generated by 3 NaBH4 + 4 BF3 2 B2H6 (diborane)+ 3 NaBF4

4 BH3 (borane) Addition of BH3 to C=C, CC is termed hydroboration. H RCH=CH2 + BH3 R H C B C H H H H H R H C B C H H H H H R C H B C H H H H

Note B is the electrophile and H acts as A-. The addition of H and B occur from the same face of the alkene (syn addition). Further reaction of RCH2CH2BH2 with more alkene yields first the dialkyl borane and then the trialkyl borane as the stable product. RCH=CH2 + RCH2CH2BH2 ( RCH2CH2)2BH RCH=CH2 ( RCH2CH2)3B

Brown discovered that trialkylboranes were easily oxidized by alkaline hydrogen peroxide (H2O2 in HO). The oxidation reaction proceeds as shown.

Chapter 6-10

Electrophilic Addition: Hydroboration

Chem 61

( RCH2)3B

OOH RCH2

CH2R BO OH

RCH2 RCH2 RCH2 B O CH2R + OH OCH2R RCH2 B O CH2R

OOH

CH2R RCH2CH2 BO O OH CH2R

OOH RCH2O

OCH2R B O CH2R

OH

3 RCH2OH + B(OH)3 The overall reaction is RCH=CH2 1. BH3 2. H2O2, OH Note the reaction is regioselective and yields an alcohol that is isomeric to that obtained by Hg(OCOCH3)2NaBH4 or H2SO4H2O. H3O+ or Hg(OCOCH3)2 then NaBH4

RCH2CH2OH

RCH=CH2

RCHCH3 OH

Treatment of a trialkylborane with CH3CO2D yields a monodeuterated alkane CH3CO2D

(CH3CH2)3B

CH3CH2D

Chapter 6-11

Electrophilic Addition: Hydroboration

Chem 61

Treatment of a trialkylborane with CH3CO2D yields a monodeuterated alkane

(CH3CH2)3B

CH3CO2D

CH3CH2D

Bromination of a trialkylborane yields the alkyl bromide. Br2 HO

(C6H5CH2CH2)3B

C6H5CH2CH2Br

Stereochemical Results of Hydroboration Hydroboration is regioselective and stereospecific

CH3 H

B BH3 H H

H2O2 HO

OH H H + H

OH H

CH3 cis-addition

CH3 CH3 enantiomers

Thus H and OH add cis to the C=C.

Chapter 6-12
Stereochemistry of Halogenation Br + C C Br

Electrophilic Addition: Stereochemistry of Halogen Addition

Chem 61

Br C C Br

C C

Alkenes containing alkyl substituents react with Br2 via a bridged bromonium ion intermediate. Nucleophilic opening of the bridged ion by backside attack of Br at carbon gives overall anti (trans) addition of E+(Br+) and A (Br) to the double bond. Chlorination reactions proceed by a similar pathway.

Some examples: CH3 Br2 H or Cl2 X CH3 C H C CH3 H + X (enantiomers) CH3 H X C C H X CH3

CH3 C H

CH3CO2H (solvent) X CH3 C H C H CH3 Br2 or Cl2 CH3

C H

H CH3 meso C X

Br2 or Cl2 CCl4 X

X H H +

X HH X

Chapter 6-13
Mixed Addition

Electrophilic Addition: Mixed Addition

Chem 61

Br C C Br+ Br+ C C Cl

Br C C Br Br C C Cl

Br2 or Cl2 with H2O, HO present:

C C

X+

X+ C C

HO

X C C OH

Mixed Reagents R S+ RSCl + C C C C NO C I ICl + C C C


+

RS C C Cl NO C Cl I C C Cl C

O=NCl +

+ C

Chapter 6-14
Hydrogenation of Alkenes addition of H2 is cis (syn) H H2/Pd H CH3 CH3 H2/Pd CH3 H H C C CH3 CH3

Hydrogenation of Alkenes
Alkene Heat of Hydrogenation, Hh (kcal/mol) CH3CH2CH=CH2 -30.3 CH3CH=CHCH3, Z -28.6 CH3CH=CHCH3, E -27.6 Thus the stability of the three alkenes is

Chem 61

CH3

CH3CH=CHCH3, E > CH3CH=CHCH3, Z > CH3CH2CH=CH2 These comparisons indicate i) increasing alkyl substitution stabilizes an alkene ii) conjugated dienes are more stable than non-conjugated dienes, iii) trans alkenes are more stable than cis alkenes (steric repulsions in cis)

CH3 C C CH3

ii) Heat of Hydrogenation Studies The heat of hydrogenation of an alkene is the energy difference between the starting alkene and the product alkane. It is calculated by measuring the amount of heat released in a hydrogenation reaction. H H C CH3CH2 H C CH3 H C CH3 C H Z-CH3CH=CHCH3 E-CH3CH=CHCH3 C CH3 CH3 C H H CH3CH2CH2CH3 + heat

CH3CH2CH=CH2

Hh

Hh

Hh progress

progress

progress

Chapter 6-15

Radical Addition Reactions

Chem 61

Addition of HBr in the presence of a radical initiator such as peroxides effects so called anti Markovnikov addition of HBr CH3 C CH2 + HX + C X CH3 CH3 X C CH3

Reversal of regioselectivity (position of bromine and H addition) results from the inital addition of Br radical rather than H+.

CH3

CH3

CH3

CH3

Stability of Carbon Free Radicals (Which H atom is abstracted?) Stability of carbon free radicals follows the same pattern as carbocations:
.

Free Radical: any atom or group of atoms that contains one or more unpaired electrons. A free radical is symbolized as a single dot representing the unpaired electron: Cl , Br , H3C , etc. Free radicals are usually encountered as high energy short-lived, non-isolable intermediates in certain reactions.
. .

CH2=CHCH2 , C6H5CH2 > (CH3)3C > (CH3)2CH > CH3CH2 > CH3

.
allyl

benzyl

tertiary

secondary

primary

methyl

heat or light ROOR peroxide RO

2 RO .

ROOR initiation R C R CHR + HBr R

.
C

Br CHR

H R C R

Br CHR

.
C

+ HBr

ROH + Br

propagation

R 3 carbocation more favored

CH3

CH2 + Br

.
CH3 C

CH3 combination of any two radicals Br

Br HBr CH2 CH3

H C

Br CH2 + Br

CH3

CH3 propagation

. + Br . . Br
C CH2 + Br CH3

Br2 termination

CH3

.
ROOR

CH3

Br Br C CH2 CH3

2 RO

Chapter 7-1
Free Radical Substitution Reactions

Radical Reactions of Alkanes

Chem 61

Free Radical: any atom or group of atoms that contains one or more unpaired electrons A free radical is symbolized as a single dot representing the unpaired electron: Cl , Br , H3C , etc. Free radicals are usually encountered as high energy short-lived, non-isolable intermediates in certain reactions. (Recall that carbocations were seen to be intermediates in certain reactions in Chapter 5). CH3 a) CH3 C Br CH3 CH3
C+ Br

carbocation intermediate

CH3 double barbed arrows indicate the movement of two electrons CH3 b) CH 3 C CH3 Br CH3 CH3 C

CH3

Br.

free radical intermediate

CH3

single barbed arrows indicate the movement of one electron Note that the carbon in a) has lost an electron pair to the bromine atom, thus the carbon has an sp2 configuration and a single "+" charge resulting from the empty p orbital. In b) the carbon atom has lost one electron; the carbon is also in the sp2 configuration with the unpaired electron in the p orbital.

CH3 CH3

CH3

(CH3)3C+

CH3 CH3

.
C CH3

(CH3)3C

equal probability of electron being in either lobe

Chapter 7-2
Free Radical Substitution Reactions Chlorination of methane in the presence of light is the classic example

Radical Reactions of Alkanes


Hence CH2Cl2, CHCl3, and CCl4 are formed c) Termination of the Chain Reaction

Chem 61

CH4 + Cl2

light

CH3Cl + CH2Cl2 + CHCl3 + CCl4 + HCl

Any reaction of Cl or of the carbon free radical intermediates (CH3 ,


.

Reaction mechanism involves three general steps: Initiation, propagation, and termination a) Initiation of the radical chain reaction heat or light

CH2Cl, CHCl2, CCl3) that disrupts the propagation of the chain will

terminate the reaction. Cl Cl

. .

+ CH3

. .

Cl Cl

2Cl requires 58 kcal/mole two chlorine radicals

CH3 + CH3

CH3Cl CH2Cl2 CH3CH3 ClCH2CH2Cl

CH2Cl

CH2Cl + CH2Cl

b) Propagation (self-perpetuating the chain reaction)

i)

Cl

. .

H H C H

HCl + CH3

requires 1 kcal/mole a methyl radical

ii)

CH3 + ClCl

CH3Cl + Cl

chlorine radical can now react with more CH4 and propogate the chain Note: Cl has substituted for a H and therefore free radical substitution iii) As the concentration of CH3Cl increases in the reaction mixture Cl starts to react with both CH4 and CH3Cl. Cl
.

+ CH3Cl

HCl +

CH2Cl (chloromethyl radical)

CH2Cl + ClCl

ClCH2Cl + Cl

Chapter 7-3
Reactivity of the Halogens F2 Bond Dissociation Energy Cl2 Br2 I2 46 36 kcal/mole
. .

Reactivity of the Halogens

Chem 61

37 58

Thus the energy of activation for formation of F and I is lower than Cl and Br however, the order of reactivity of X. with alkanes is: F2 >> Cl2 > Br2 >> I2 (I2 does not normally react; F2 reacts explosively) Thus the rate determining step is not: XX 2X.

Rather, hydrogen atom abstraction is the rate determining step.

X + CH4

HX +

CH3

Since chlorination is faster than bromination, Eact for Cl + CH4

HCl +

CH3

must be lower than the Eact for Br + CH4

HBr + CH3

[CH3 + HCl +Cl2] E Eact Cl + CH4 CH3Cl + Cl Progress

[CH3 + HBr + Br2]

Eact Br + CH4

CH3Br + Br

Chapter 7-4
Stability of Carbon Free Radicals (Which H atom is abstracted?)

Reactivity of the Halogens

Chem 61

Stability of carbon free radicals follows the same pattern as carbocations: CH2=CHCH2 , C6H5CH2 > (CH3)3C > (CH3)2CH. > CH3CH2 > CH3 allyl benzyl tertiary secondary primary methyl

Unlike carbocations, free radicals do not rearrange to a more stable free radical. CH3 CH3 C CH CH3

CH3 CH3 C CH CH3 + CH3 2 CH3 CH3 C CH CH3 Cl2 CH3 CH3

CH3 C + 3 CH CH3 CH3 CH3 C CH CH3

H 2O

OH CH3

CH3 2

CH3Cl

Since both carbocations and free radicals involve planar sp2 carbon atoms, racemization will be observed in the products resulting from both intermediates.

H CH3CH2 C CH2Cl CH3 R Br CH3CH2 CH3 C CH2Cl S CH3OH Cl2

Cl

CH3CH2 C CH2Cl CH3 S (50:50) OCH3 CH3CH2 C CH2Cl CH3 S

CH3CH2 + CH3 R

C Cl

CH2Cl

CH3CH2 + CH3

C R

CH2Cl

OCH3

(50:50)

Chapter 7-5
Selectivity of Hydrogen Atom Abstraction

Selectivity of Halogenation
Other examples of this are: Cl

Chem 61

The more stable free radical should determine the nature of the product CH3 CH2

Br.

CH3

CH3

CH3

.
d) allyl

CH3

CH2CH3 Cl2, light

CHCH3 + (56:44) Br

CH2CH2Cl

a) 1

b) 3

c) allyl

e) 2 Br2, light

Thus intermediates c) and d) should be more stable and should lead to CH3 and Br as products.

CHCH3 100%

CH3

Br

Chlorination is much less specific than bromination Cl CH3CH2CH3 Cl2 CH3 CH CH3 55% Br CH CH3 100% + CH3CH2CH2Cl 45%

CH3CH2CH3

Br2

CH3

The difference in selectivity of H atom abstraction by Cl and Br is explained by the Hammond postulate. The transition state in the chlorination reaction is less influenced by the stability of the intermediate free radical; thus both (CH3)2CH and CH3CH2CH2 result.

In bromination the more stable free radical intermediate is highly favored; thus (CH3)2CH is formed exclusively.

Chapter 7-6
Bromination with N-Bromosuccinimide (NBS) O CCl4 NBr + O light or peroxide NH + O Br O

Halogenations

Chem 61

NBS acts as a Br2 source; the reaction is initiated by either light or a peroxide (ROOR). NBS is used to introduce a Br atom at allylic or benzylic positions.

Other Sources of Free Radicals Although light (h) and heat () are used for halogenation reactions other reagents are often useful for initiating free radical reactions. Some of these reagents include: a) dibenzoyl peroxide C6 H5 C O b) hydrogen peroxide H O O H 2 H O. O O C O C 6H 5 2 C6 H5 C O O

c) alkyl hypochlorites ROCl d) azobisisobutyrylnitrile CH3 CH3 C N N

RO + Cl

CH3 C CH3 2 CH3

CH3 C

+ NN

CN.

CN.

CN.

Clues to Whether a Reaction Involves Free Radicals a) Reaction requires high temperatures (>200) b) Reaction requires light energy (h) c) Reaction requires an initiator (a-d above) or oxygen.

Chapter 8-1
Nucleophilic Substitution Reactions (SN1, SN2) alkyl halides: CH3X methyl X aryl halide vinyl halide CH2=CHX RCH2X primary, 1 R2CHX secondary, 2

Nucleophilic Substitution Reactions

Chem 61

R3CX tertiary, 3

do not easily undergo nucleophilic substitution reactions

Examples CH3I + Na+ OCH3 CH3CH2CH2Br + Na+ Cl + Na


+

CH3OH CH3OH

CH3OCH3 + Na+

OH

CH3CH2CH2OH + Na+ Br SH + NaCl

SH

CH3OH

Br + CH3 CH3OH

OCH3 + HBr CH3

Anatomy of a nucleophilic substitution reaction

SH CH3 CH I CH3 + Na+ SH CH3OH CH3 CH CH3 + Na+ I

Chapter 8-2
Anatomy of a nucleophilic substitution reaction

Nucleophilic Substitution Reactions

Chem 61

CH3

CH I

CH3

+ Na+ SH

CH3OH

CH3

CH SH

CH3

+ Na+

(L) Leaving Group: any group that can be displaced from a carbon atom (in this case I) (Nu) Nucleophile: the species that attacks the carbon atom bearing L and donates the electron pair to form the nucleophileC bond (in this case SH) (RI) Substrate: the molecule, containing the leaving group that is acted on by the nucleophile (here: 2-iodopropane) Solvent: the medium used to dissolve the substrate and the nucleophile (in this case methanol, CH3OH); solvent can sometimes be the nucleophile (solvolysis) Curved arrows are used to indicate the movement of electrons during a nucleophilic substitution reaction. By convention electron movement is written from negative to positive. + Nu: + RCH2L electron pair that accompanies L RCH2Nu + L:

electron pair for new bond at C

Chapter 8-3

Nucleophilic Substitution Reactions


Energetics of SN2

Chem 61

Reaction Mechanisms of Nucleophilic Substitution Reactions

reaction mechanism: a detailed description of how a chemical reaction occurs. A roadmap of a reaction....curved arrows show which bonds are formed or broken. A mechanism includes the transition states involved in making and breaking bonds and reactive intermediates that are formed along the pathway from reactants to products. potential energy E CASE 1:SN2 Substitution Nucleophilic 2nd order (bimolecular): sp3 CH3 CH3O

energy of transition state

Eact

sp2 sp3 CH3 Br CH3O C H partial bonds in transition state H Br CH3O C + Br CH3 H H

reactants products progress of reaction

H of reaction

H H

Orbital Picture H Nu Nu nonbonding orbital H H Nu C H + L L nonbonding orbital H C L Nu H H C H L

H CL sigma*

NuC sigma

Chapter 8-4
Orientation of Nu: and L in SN2: Nucleophile approaches substrate from the backside of L

Nucleophilic Substitution Reactions

Chem 61

Point of highest energy along the reaction coordinate is the transition state: the OC bond is partially formed and the CBr bond is partially broken The Nu is bonded to the carbon on the opposite side of that occupied by the leaving group L: Net inversion of the carbon atom is observed in all SN2 reactions Reaction Rate: the time required for all substrate molecules to be converted to product The rate of an SN2 reaction is proportional to the concentrations of the substrate and the nucleophile, thus the reaction is second order Rate = k[substrate][nucleophile] k is the proportionality constant called the rate constant Rate and Eact Under the same reaction conditions, the reaction with the lower Eact has a faster rate Reaction 2 below is faster

Eact E

Eact

Rxn 1

Rxn 2

Increasing alkyl group substitution at the carbon atom bonded to the leaving group hinders approach of the nucleophile. The E increases due to steric hindrance of act nucleophile approach and thus the SN2 rate decreases

Chapter 8-5
Relationship between Substrate Structure and SN2 Rate Alkyl halide Relative rate of SN2 CH3X 30 CH3CH2X 1 CH3CH2CH2X 0.4 (CH ) CHX 0.025 3 2 (CH ) CX ~0 3 3

Nucleophilic Substitution Reactions

Chem 61

Relative SN2 rate CH3Cl + I

CH3I

Cl

93 1 0.0076

CH3CH2Cl + I (CH ) CHCl + I 3 2

CH3CH2I + Cl (CH ) CHI + Cl 3 2

Increasing alkyl group substitution at the carbon atom bonded to the leaving group hinders approach of the nucleophile. The E increases due to steric hindrance of act nucleophile approach and thus the SN2 rate decreases SUMMARY SN2 reactions occur by the attack of a nucleophile on substrates containing the leaving group attached to a methyl, primary carbon, or secondary carbon. SN2 reactions do not occur with tertiary alkyl halides SN2 exhibit a single transition state (concerted reaction, i.e. not stepwise, no intermediates) Inversion of configuration at the carbon results from backside attack of the nucleophile The rate of SN2 reaction depends on the concentration of the nucleophile and the substrate The rate of SN2 follows the order: CH3X > CH3CH2X > CH3CH2CH2X > (CH) CHX >> (CH 3CX 3 2 3)

Chapter 8-6

Nucleophilic Substitution Reactions

Chem 61

CASE 2: SN1: Substitution Nucleophilic First Order (Unimolecular) Substrates containing the leaving group attached to a tertiary carbon atom react with weakly basic nucleophiles by an alternative nucleophilic substitution reaction mechanism (CH3)3CBr + CH3OH The process involves three steps: Step 1: ionization (loss of halide ion) (CH3)3CBr + CH3OH slow 1 Step 2: attack by the nucleophile .. CH3OH .. + (CH3)3C Br fast 2 Step 3: deprotonation (loss of proton) (CH3)3C O+ CH3 H Br Step 1 involves ionization of the alkyl halide to the halide ion and the tertiary carbocation. The step with the highest Eact is the slowest step (rate determining step) in the pathway. Ionization is aided by polar solvents (H2O, ROH) which solvate and thus stabilize the carbocation and the leaving group anion. Step 3 involves acid-base reaction between either Br or more likely solvent CH3OH in a very rapid reaction fast 3 (CH3)3C O CH3 + HBr (CH3)3C+ + Br The transition state is pictured as (CH3)3C+ ------Br

(CH3)3COCH3 + HBr

(CH3)3C

O+ CH3 H

The transition state is pictured as (CH3)3C+------+OCH3 H CH3 The transition state is pictured as (CH3)3C O
+

H H O CH3

Chapter 8-7
Stereochemistry of SN1 reaction

Nucleophilic Substitution Reactions

Chem 61

Relationship Between Substrate Structure and SN1 Rate Relative SN1 rate 1.0* 1.0* 11.6 1.2 X 106

the intermediate carbocation contains an sp2 hybridized carbon atom which is planar Alkyl halide

+
CH3 CH3 C CH3

CH3Br CH3CH2Br (CH3)2CHBr (CH3)3CBr

Thus attack of CH3OH on the carbocation can occur equally from the top and bottom. If the carbon atom containing the leaving group were asymmetric, racemization of the product would be observed Br CH3 CH3CH2 R Reaction Rate of SN1 The rate of the SN1 reaction does not depend on the concentration of the nucleophile, but depends only on the concentration of the substrate SN1 Rate = k[substrate] The reaction is first order because the rate is proportional to the concentration of only one reactant C CH2CH2CH3 H 2O CH3CH2 CH3

*These observed reaction rates probably proceed through SN2 not SN1 SN1 rates reflect the relative Eact leading to the different carbocations (transition state 1)

OH CH3 CH3CH2 C R + CH2CH2CH3

CH2CH2CH3

(50:50)

OH S

Since the relative stability of carbocations is 3 > 2 > 1 > methyl; tertiary alkyl halides proceed at the faster rate

Chapter 8-8
Allyl and benzyl primary halides

Nucleophilic Substitution Reactions


SUMMARY

Chem 61

Allyl and benzyl primary halides are reactive substrates in both SN1 and SN2 type reactions. Substrate CH3X CH3CH2X CH2=CHCH2X C6H5CH2X Realtive SN1 Rate 1.0 1.0 33 380 Relative SN2 Rate 30 1 40 120

SN1 reactions involve an intermediate carbocation which is attacked by the nucleophile to yield the product of nucleophilic substitution. SN1 reactions occur with substrates that yield stabilized carbocations (allyl, benzyl, 3, 2, not primary or methyl) SN1 reactions are first order in rate. SN1 reactions lead to racemization.

Reactions of Carbocations 1. Reaction with a nucleophile: SN1 reactions R+ + Nu: or Nu: 2. Elimination of H+ on an adjacent carbon: E1 reactions CH3 + CH CH2 H 3. Rearrangement to a more stable carbocation: occurs whenever a more stable carbocation can be formed CH3 + CH H C CH3 CH3 CH3 + CH2 C CH3 CH3CH=CH2 + H+

CH3

Chapter 8-9
Elimination Reactions (E1, E2) Case 1: E1: Elimination First order

Elimination Reactions

Chem 61

Formation of a carbocation intermediate in a reaction is a first order process. Once formed the carbocation may react with a nucleophile (SN1) or lose a proton from an adjacent carbon to form an alkene (E1) Thus SN1 and E1 reactions are competitive CH3 C + Br H (CH3)3C
Nu +Br (SN1)

(CH3)3CBr

CH3

H C H

CH3 C CH3

CH2

+ HBr

(E1)

E1 reactions predominate when the reaction contains only poor nucleophiles; otherwise SN1 reactions are the more likely pathway Since a carbocation is involved, rearangement may occur

TS1 TS2 E Eact R+ RX alkene

Chapter 8-10
CASE 2: E2: Elimination Second Order

Elimination Reactions
Thus CH3CH2CH2 CHCH3 Br RO 80, ROH

Chem 61
CH3CH2CH 69% + CH3CH2CH2CH 31% CH2 CHCH3

Bimolecular elimination results when an alkyl halide is treated with a strong base (HO , RO , etc) at elevated temperatures (80-120). The E2 reaction does not involve an intermediate as in the E1 reaction E2 reactions are concerted as in the SN2 reaction H RO H C H H C H Br ROH + H H C C H H + Br

If RO is bulky (CH3)3CO vs. CH3O; a higher percentage of the less substituted alkene results

RO H H H C

becoming more sp2 H H

RO H H C H C H H Br CH3CH2 CHCH3 Br

CH3O

CH3CH

CHCH3 +

CH3CH2CH 20% + CH3CH2CH 50%

CH2

80% (CH3)3CO CH3CH CHCH3 50%

CH2

Br

transition state for E2 Elimination Reaction rate E2 rate = k[base][substrate] halides Direction of Elimination The more substituted (and more stable) product is normally the predominant E2 product CH2=CH2 RCH=CH2 RCH=CHR diR2C=CH2 diR2CH=CHR triR2C=CR2 tetratertiary halides > secondary halides > primary

unsubstituted monoincreasing alkene stability

Chapter 8-11
Stereochemistry of E2

Elimination Reactions
SUMMARY

Chem 61

Generally, in E2 reactions the proton and the leaving group must be in an anti orientation. This allows for the best overlap of the developing pi orbitals.

E1 reactions result from carbocation intermediates and are competivive with SN1 reactions. E1 reactions may be accompanied by carbocation rearrangements.

RO H b c L In order to predict the stereochemistry of the alkene resulting from an E2 reaction: a) rotate the C atoms so that the H to be removed by RO and the leaving group are in the anti conformation b) the stereochemistry of the alkene resulting from the E2 reaction is as shown a d b C a C d c + ROH + L

E1 reactions generally yield the most substituted (most stable) alkene. E1 reactions are not stereoespecific and yield a mixture of E,Z alkenes. Relative rates: 3 > 2. E2 reactions result from a concerted reaction and are competitive with SN2 reactions. E2 reactions generally produce the more stable alkene (except with bulky bases or leaving groups). E2 reactions are stereospecific and give anti elimination of H and L. Relative rates: 3 > 2 > 1.

H Ph CH3 C C H

Br H rotate Ph C Ph CH3

RO Ph H Br Ph C CH3 Z C H Ph

In cyclohexane systems the H and the leaving group must be trans and diaxial Cl D H H D H

RO

Chapter 8-12

Substitution vs. Elimination Reactions


Temperature

Chem 61

Factors Governing Substitution and Elimination Reactions Structure of Alkyl Halide Leaving group attached to methyl or primary carbon: SN2 reaction observed unless Nu: is a strong base and elevated temperature employed; then observe E2. Leaving group attached to tertiary carbon: E2 reaction observed unless Nu: is weak base; then SN1 observed. Solvent nucleophilicity increases with increasing electron releasing capacity of molecule. If the solvent has low nucleophilic properties; i.e. H2SO4, H3PO4 then E1 reaction results CH3CH2OH > H2O > CH3CO2H > CF3CH2OH > CF3CO2H increasing solvent nucleophilicity Leaving group attached to secondary carbon: if the nucleophile is less basic than HO (e.g. CN, N3, SR) then the reaction will follow SN2 path If the Nu: is HO or more basic (e.g. RO, R2N, RCC), then the reaction will follow E2 path If the Nu: is a weak nucleophile (e.g. H2O, ROH, RCO2H) the reaction will follow SN1 path with some E1 The leaving group is attached to an allylic or benzylic carbon: SN1

Increase in temperature increases the rates of all reaction types but has a larger effect on E2 reactions.

Solvent Highly polar solvents (high dielectric constant) favor SN1, E1 reactions Concentration of Nucleophile or Base Increasing the concentration of the nucleophile has no effect on SN1, E1 but increases SN2,E2 rates.

Chapter 8-13
NUCLEOPHILICITY Nucleophilic Constants of Various Nucleophiles Nucleophile CH3OH NO3 F CH3CO2 Cl (CH2)2S NH3 N3 PhO Br CH3O HO NH2OH NH2NH2 (CH3CH2)3N NC I HOO (CH3CH2)3P PhS PhSe Ph3Sn n(CH3I)a 0.0 1.5 2.7 4.3 4.4 5.3 5.5 5.8 5.8 5.8 6.3 6.5 6.6 6.6 6.7 6.7 7.4 7.8 8.7 9.9 10.7 11.5 pKa of conjugate acid -1.7 (CH3OH2) -1.3 3.45 4.8 5.7 9.25 4.74 9.9 -7.7 15.7 15.7 5.8 7.9 10.7 9.3 -10.7 8.69 8.5

Nucleophilicity
General comments

Chem 61

1. Note that nucleophilicity toward CH3I does not correlate directly with basicity. (N3, PhO, Br- are equivalent nucleophilicity but differ greatly in basicity. Also N3 and CH3CO2 are nearly identical in basicity but N3 is 30 times [1.5 log units] more nucleophilic). 2. Among neutral nucleophiles while (CH3CH2)3N is more basic than (CH3CH2)3P (pKa 10.7 vs. 8.69) the phosphine is 100 times (n = 8.7 vs 6.7) more nucleophilic. 3. Correlation of nucleophilicity with basicity is better if the attacking atom is the same. Thus CH3O > PhO > CH3CO2> NO3 4. Nucleophilicity usually decreases going across a row in the periodic table. (HO > F ; PhS> Cl). This order is determined by electronegativity. 5. Nucleophilicity usually increases going down the periodic table (I > Br > Cl > F; and PhSe > PhS > PhO) Related to weaker solvation and greater polarizability of the heavier atoms.

an(CH3I) = log(knucleophile/kCH3OH) in CH3OH 25C n = nucleophilic constant

Chapter 8-14
Examples Nu: 1 R CH CH2 H hindered strong base heat R L R CH CH2 H Nu

Nucleophilicity E1: 3 > 2 > 1 E2: 3 > 2 > 1 SN1: 3 > 2 > 1 SN2: 1 > 2 > 3

Chem 61

SN2 (inversion

CH CH2

E2 (anti elimination of HL)

NuH C H C L Nu: acts as base C C E2 C C C E1

C H

C +

C H
+

Nu+ H H+

C H

C Nu S N1

H S N2 2 H C H C L poor Nu or base H C H C + C SN1 or E1 C E2 Nu: or base:


C H C

C Nu H C

Chapter 9-1
Alcohols are similar to water: contain sp3 hybridized oxygen, OH functional group bonded to an sp3 carbon, with two lone pairs on oxygen H .. O .. ethanol H CH3CH2 .. O .. H

Alcohols

Chem 61

water

Alcohols can hydrogen bond thus their boiling points are higher than similar polar compounds which cannot hydrogen bond. Low molecular weight alcohols are soluble in water Alcohols contain a hydrophobic alkyl group and a hydrophilic hydroxyl group As the alkyl group increases in length and size, the solubility in water decreases Acidity and Basicity

alcohols act as bases in the presence of strong acids CH3CH2 base H H+ acid CH3CH2 H

. .O . .

..O+ H

and as acids in the presence of strong bases

(CH3)3C

. .O . .

+ K+H base

.. (CH3)3C O. K+ + H2 . ..

acid

CH3CH2OH + Na0

CH3CH2O Na+ + H2

Chapter 9-2
IUPAC Names For alcohols drop e from the alkane name and add ol CH3CH2OH: ethan + ol = ethanol CH3CH2CHOHCH3: butan + ol; OH on carbon 2: 2-butanol Classification Alcohols are classified as: methyl, primary, secondary, tertiary, allylic, benzylic CH3OH methyl CH3CH2CH2CH2CH2CH2OH primary 1 CH3CHOHCH3 secondary 2

Alcohols

Chem 61

(CH3)3COH tertiary 3

CH2=CHCH2OH allylic

CH2OH

benzylic

REACTIONS OF ALCOHOLS Substitution Reactions: Reaction with Hydrogen Halides (HX) Alcohols do not undergo nucleophilic substitution by X since HO is a poor leaving group. Alcohols do undergo substitution by X in acidic solution. Here the alcohol is protonated and H2O, a neutral species, is the leaving group. .. OH .. H+ R .. + O H H X

RX + H2O

Chapter 9-3
Reactivity of Hydrogen Halides The reactivity of hydrogen halides in alcohol substitution reactions is as follows: HF < HCl < HBr < HI pKa 3.45 -7 -9 -9.5 The reactivity is explained since the acidity of HX increases and the nucleophilicity of X increases going from HF to HCl to HBr to HI.

Alcohols

Chem 61

CH3CH2CH2 CH3CH2CH2 OH

O H H

Reactivity of Alcohols Toward HX

CH3CH2CH2

Br

progress of reaction methyl primary secondary tertiary allylic and benzylic increasing reactivity of ROH toward HX

All alcohols react readily with HBr and HI; 3, allylic and benzylic react rapidly with HCl 2 and 1 alcohols require the addition of ZnCl2 for rapid reaction with HCl Mechanism of Alcohol Substitutions Methyl and primary alcohols follow the SN2 mechanism Step 1 protonation of the alcohol CH3CH2CH2 OH H+ CH3CH2CH2 + O H H Step 2 SN2 dsiplacement of Water

CH3CH2CH2

O+H H

Br S N2

CH3CH2CH2

Br + H2O

Chapter 9-4
Secondary and tertiary alcohols follow the SN1 mechanism Step 1 protonation of the alcohol CH3 CH3 C OH Step 2 loss of water to for the carbocation CH3 CH3
+

Alcohols

Chem 61

H+ CH2CH3 CH3
+

CH3 C OH2 CH2CH3

C OH2

CH2CH3

H 2O

CH3 CH3 C + CH2CH3

Step 3 attack of halide ion on the carbocation CH3 CH3 C + CH2CH3 X S N1 CH3 CH3 C I CH2CH3

Rearrangement can occur

R+ E RO+H ROH H RX

Chapter 9-5

Conversion of Alcohols to Alkyl Halides

Chem 61

Other Reagents for the Conversion of Alcohols to Alkyl Halides ROH + SOCl2 ROH + PBr3 RCl + HCl + SO2 RBr + HOPBr2

H CH3CH2 C OH Cl CH3

O S Cl Cl

CH3CH2 C O+ S Cl CH3 Cl H H O Cl H C CH3CH2 CH3

H + CH3CH2 C OSOCl CH3 H R3N:

CH3CH2 C O S Cl CH3

+ SO2

Esters of Alcohols Reaction of alcohols with carboxylic acids in the presence of acid produces esters of carboxylic acids

O CH3CH2CH2 C OH + CH3CH2CH2OH alcohol H+ heat O CH3CH2CH2 ester C OCH2CH2CH3

carboxylic acid

Chapter 9-6
Inorganic esters

Reactions of Alcohols

Chem 61

mineral acids such as H2SO4, HNO3 and H3PO4 form esters with alcohols to produce important compounds H H H H C C C H nitroglycerin, a nitrate ester ONO2 ONO2 ONO2 H H H H C C C O H H dimethyl sulfate, a sulfate ester O P OH O O P OH OH CH3O

O S O OCH3

H a diphosphate ester

P-toluenesulfonates (tosylates) and methanesulfonates (mesylates) are excellent leaving groups in nucleophilic substitution reactions. They are readily prepared form an alcohol and the corresponding sulfonyl chloride. O CH3O S O a methanesulfonate (mesylate) CH3 + (CH3CH2)3N+HCl

CH3OH + ClSO2CH3

(CH3CH2)3N

O OH
+

O CH3 S Cl O
p-toluenesulfonylchloride (tosyl chloride)

S O

CH3

cyclohexyl tosylate

Reaction of sulfonates with nucleophiles CH3CH2CH2O Na+

OTs

OCH2CH2CH3

Chapter 9-7
Elimination (Dehydration) of Alcohols Alcohols undergo elimination much like alkyl halides

Dehydration of Alcohols
Examples: CH2 CHCH2CH3 OH

Chem 61

H+

CH

CHCH2CH3

Tertiary alcohols readily undergo dehydration by an E1 pathway Secondary alcohols also follow an E1 path, but primary alcohols probably eliminate by an E2 mechanism conc. H2SO4 (CH3)3COH ease of dehydration (CH3)2CHOH CH3CH2CH2OH Mechanism: H CH3 C CH3 H CH3 C CH3 CH3 C CH3 CH3 C+ CH3 CH3 OH H+ CH3 H C CH3 CH3 C CH3 O+H2 H 2O CH3 60 C conc. H2SO4 100 C conc. H2SO4 180 C (CH3)2C=CH2 +H2O CH3CH=CH2 +H2O CH3CH=CH2 +H2O CH3 CH3 C CH3 CH3 + C CH3 H+ CH3 C C CH3 CH3 CH3 CH3 C C CH3 C H CH3 CH3 1,2 shift CH3 + C CH3 CH3 C H OH H+ CH3 CH3 C CH3 CH3 C H CH3 C H CH3 O+H2 OH CH CH3 CH3

H2SO4 heat

CH3 CH3

H 2O

H+

The most stable alkene predominates in dehydration reactions. Rearrangements can occur.

E ROH2+ ROH

R+

alkene

progress of reaction

Chapter 9-8
ETHERS, EPOXIDES AND SULFIDES

Dehydration of Alcohols

Chem 61

Ethers are derivatives of water where both hydrogens have been replaced by an alkyl group. Ethers are less polar than alcohols and water and are not capable of hydrogen bonding to themselves because of the lack of an OH group. The boiling points of ethers are also much lower than alcohols of comparable molecular weight.

CH3CH2OCH2CH3 O diethyl ether Preparation of Ethers Williamson Ether Synthesis (SN2 Reaction of an Alkoxide with an Alkyl Halide) tetrahydrofuran CH2

O CH2 ethylene oxide

R1O + R2X

S N2

R1OR2 + X

Best results are obtained if the alkyl halide is methyl or primary (2 and 3 give mostly elimination) There are few limitations on the alkoxide CH3CH2CH2O + (CH3)2CHCH2Br CH3CH2CH2OCH2CH(CH3)2

O + CH3CH2I

OCH2CH3

(CH3)3CO + CH3I

(CH3)3COCH3

Chapter 9-9
Substitution Reactions of Ethers

Reactions of Ethers and Epoxides

Chem 61

Ethers are relatively unreactive compounds, but they do undergo substitution reaction when heated with hydrogen halides, particularly HI and HBr. This is a very similar reaction to the reaction of alcohols with HX Br CH3CH2CH2OCH2CH2CH3

.. ..

HBr

+ CH3CH2CH2OCH2CH2CH3 H HBr

CH3CH2CH2Br + CH3CH2CH2OH

CH3CH2CH2Br

OCH2CH(CH3)2

HI

OH + ICH2CH(CH3)2 unreactive with HI

Reactions of Epoxides Epoxides are strained much like cyclopropanes because their bond angles (60) are far removed from the normal tetrahedral angle (109.5). The orbitals have poor overlap and the bonds are weakened. The CO bond is also polarized and consequently, epoxides are highly reactive compounds. Base Catalyzed Cleavage of Epoxides Alkoxides: CH3 C CH3 O C H CH3OH H CH3O

H C H OCH3

HOCH3 CH3

HO C H 3C

H C H OCH3

CH3 C H 3C

CH3O nucleophile attacks at the least hindered carbon in an SN2 reaction

Chapter 9-10
Grignard Reagents O C H C H CH3MgBr BrMgO H C H H C H CH3 H+ H

Reactions of Epoxides

Chem 61

HO C H

H C H CH3

CH3MgBr+

Acid Catalyzed Cleavage


+ CH3 O H H C C H CH3OH CH3

H + O CH3 + C CH3 H C H .. CH3OH .. CH3OH CH3 C CH3O C H H

CH3OH CH3 C C H CH3O + H H

H+

Nucleophile attacks at the most hindered carbon since the carbon is partially positive and the CO bond is partially broken H
+

O H

OH H H OH trans diaxial opening in cyclohexanes

H H 2O

Chapter 9-11
Crown Ethers

Crown Ethers and Thiols

Chem 61

Crown ethers are macrocyclic ethers with repeating OCH2CH2 units. Depending on the ring size, they effectively chelate alkali metal ions such as K+, Na+ or Li+ O O O O O Na+ O K+ O O O O O 18-crown-6 Thiols and Sulfides The sulfur analog of an alcohol is a thiol, or mercaptan. Thiols are stronger acids than alcohols (pKa = 8, ROH = 16). Thiols form weaker hydrogen bonds than alcohols due to the lower electronegativity of sulfur. Thiols and thioethers (sulfides) are prepared by substitution reaction in the same way that alcohols and ethers are prepared. Br HS SH 12-crown-5

CH3CH2CH2S I SCH2CH2CH3

Disulfides are formed by the oxidation of thiols and are an important structural feature in some proteins. RSH I2 or K3Fe(CN)6 RSSR

a disulfide

Chapter 9-12
Grignard Reactions

Organometallic Compounds

Chem 61

Grignard reagents are organometallic reagents derived from an alkyl halide and magnesium RX + Mg diethyl ether RMg+X Grignard reagent

Since the carbon carries a partial negative charge, the carbon is a strong base and a good nucleophile. diethyl ether CH3CH2Mg+Br HOH CH3CH2H

CH3CH2Br + Mg

Because carbonyl pi bonds are polarized, they can undergo a reaction called nucleophilic addition: the addition of a nucleophile to an electron deficient pi bond. O C+ R1 R1 Nu: R1 O C+ R1 R1 O C Nu R1 Nucleophilic Addition

O C+ R1 R1 R2 A Grignard reaction with R1

O C+ R1 R1

O MgX+ C R2 R1

Nucleophilic Addition

Mg-X

1. formaldehyde produces a primary alcohol O C H H 1. CH3 Mg-X H OH C CH3 H

2. H2O, H+

Chapter 9-13

Organometallic Compounds

Chem 61

2. an aldehyde produces a secondary alcohol

O C CH3CH2 H 1. (CH3)CHMgBr 2. H2O, H+ H

OH C CH2CH3

CH(CH3)2

3. a ketone produces a tertiary alcohol O C CH3CH2 OH CH3 C CH2CH3

1. (CH3)CHMgBr
+ CH3 2. H2O, H

CH(CH3)2

4. an ester produces a tertiary alcohol (addition of two molecules of Grignard reagent) O C OCH3 1. 2 CH3MgBr 2. H2O, H+ CH3 OH C CH3

5. ethylene oxide produces a primary alcohol

1. C6H5MgBr 2. H2O, H+ OH

Chapter 10-1
sp Hybridization sp hybridized carbons are linear with atoms 180 apart

Alkynes: Structure and Bonding

Chem 61

2p 2p 2s 1s 96 kcal 2p 2s 1s hybridize 1s sp

ethylene: trigonal planar + 2s 180 apart; the remaining two 2p orbitals are 90 to the sp and each other

+ 1-2p 2-sp's

Acetylene: linear; bond angles 180 C=C bond length = 1.20 acetylene has two perpendicular pi bonds and one sigma bond

H
2-sp's

acetylene -orbital

2-2p's on each carbon combine to form pi-orbitals CC * CC * CC CC

acetylene -orbitals

Chapter 10-2
Reaction of Alkynes with E+A Reagents

Alkynes: Electrophilic Addition

Chem 61

Reaction of alkynes with E+A reagents proceed in the same manner as alkenes except different intermediates are possible

E+

+ C

C E+

E R

vinyl carbocation

C R

bridged intermediate

For HX the vinyl carbocation is more stable than the bridged intermediate. Thus:

E+

+ C

C E+

E R

vinyl carbocation

C R

bridged intermediate

secondary vinyl carbocation is about the same stability as a primary carbocation Thus the order of carbocation stability is triphenyl methyl > diphenylmethyl > 3 benzyl allyl > 2 > 1 2 vinyl >> methyl

Chapter 10-3
For HX HCl CH3 C C H Cl HCl CH3 C Cl CH3 C C H H

Alkynes: Electrophilic Addition

Chem 61

CH3

C CH3

CH3 CH3 C C + H Cl

H C CH3

For Hg(OCOCH3)2: Hg(OCOCH3)2 CH3CH2 C CH3CO2 E C CH2CH3 HgOCOCH3

C2H5CCC2H5

For Cl2, Br2: X2 RCCR R but, C6H5CCC6H5 X2 C X+ C R X C C 6H 5 X C C6 H5 C X X X C C6 H5 + C C 6H 5 C 6H 5 R C C R X X

+ C C6 H5

Thus the choice of intermediate depends on structure; alkyl groups tend to favor the bridged ion; groups such as phenyl which stabilize the free carbocation tend to proceed via the vinyl carbocation.

Chapter 10-4
Addition of Water

Alkynes: Electrophilic Addition

Chem 61

With alkynes this electrophilic addition reaction generates a vinyl alcohol (also called an enol). Hg(II) ion is often used. H2SO4 Hg(II) H 2O OH RC CHR enol O RC CH2R keto

RC

CR

H2O: RC CR Hg2+

H +OH RC CR Hg2+ H+

H2O:

RC

OH CR

H O RC CR Hg2+

Hg2+

RC H

OH CR

H O RC CR H

Hydroboration H RC CH BH3 RC BH2 CH H RC OH CH

enol H O RC H CH keto (aldehyde)

Chapter 10-5
H C C + base C H pKa = 45 C C H + base C . C. .. C H

Alkynes: Acidity
HF + baseH pKa 3.2 H 2O 15.7 HCCH 25 NH3 36 H2C=CH2 44

Chem 61
H3CCH3 50

increasing acid strength + baseH F HO HCC NH2 H2C=CH H3CCH2 pKa = 25 The relatively high acidity of the alkyne CCH bond is associated with the large degree of s character in the sp CH bond (50% compared with 33% in sp2 bonds). The carbon atom is more electronegative in the sp state; thus the CH bond is more acidic. The acetylide ion may be formed by such strong bases as :NH2 (pKa 33), RMgX or RLi (pKa 45-50).

increasing base strength

NaNH2 NH3

C: Na+ + NH3

acetylide ion No reaction

CH

CH2

NaNH2 NH3

Electronegativities

HCCH sp >

H2C=CH2 sp
2

H3CCH3 sp3

>

Electronegativities acid strength pKa

N < O < F NH3 < H2O < HF 36 15.7 3.2

Chapter 10-6
SN2 reaction with acetylide ion NH3 R'CC: Na
+

Alkynes: Acetylides

Chem 61

RCH2L

RCH2CCR'

R'

C: MgBr+ +

O R' C

CCH2CH2O MgBr+ H+

R'

CCH2CH2OH

Nucleophilic addition reaction with acetylide ion. O MgBr+ H R' C C H+ OH R' C C C H R C H CH3CH2MgBr R' C CH R

O R' C C: MgBr+ R C

O R' C C: MgBr
+

O MgBr+ C C R' H+ OH C

C R'

Chapter 12-1
Infrared and Nuclear Magnetic Resonance Spectroscopy

Spectroscopy

Chem 61

electromagnetic radiation: energy that is transmitted through space in the form of waves wavelength: (): the distance from the crest of one wave to the crest of the next wave frequency: (): the number of complete cycles per second

where c = speed of light

Electromagnetic radiation is transmitted in particle-like packets called photons or quanta. The energy is inversely proportional to the wavelength and directly proportional to frequency.

hc

where c = speed of light; h = Planck's constant

= h
ultraviolet visible

h = Planck's constant

infrared

radio

decreasing energy

Absorbtion of ultraviolet light results in the promotion of an electron to a higher energy orbital. Absorbtion of infrared results in increased amplitudes of vibration of bonded atoms. Intensity of radiation is proportional to the number of photons.

Chapter 12-2
Infrared Spectroscopy Infrared is recorded as %T versus wavelength or frequency

Infrared Spectroscopy

Chem 61

When a sample absorbs at a particular wavelength or frequency, %T is reduced and a peak or band is displayed in the spectrum. Infrared is recorded as %T versus wavelength or frequency When a sample absorbs at a particular wavelength or frequency, %T is reduced and a peak or band is displayed in the spectrum.

100 %T

frequency

Nuclei of bonded atoms undergo vibrations similar to two balls connected by a spring. Depending on the particular atoms bonded to each other (and their masses) the frequency of this vibration will vary. Infrared energy is absorbed by molecules resulting in an excited vibrational state. Vibrations occur in quantized energy levels and thus a particular type of bond will absorb only at certain frequencies. Both stretching and bending vibrations can be observed by infrared.

O CH3

CH3

stretching

O CH3

CH3

bending

Chapter 12-3
Interpretation of Infrared Spectra Correlation tables

Infrared Spectroscopy
CC and CH Bonds

Chem 61

Infrared spectra of thousands of compounds have been tabulated and general trends are known. Some common functional groups are shown below.

OH and NH str CH str CN str

C=O str C=N str C=C str NH bend

CO str CN str CC str CH bend OH bend

sp3 CC sp2 C=C sp2 CC (aryl) sp CC sp3 CH sp2 CH sp CH C(CH3)2

weak, not useful 1600-1700 cm1 1450-1600 cm1 2100-2250 cm1 2800-3000 cm1 3000-3300 cm1 3300 cm1 1360-1385 cm1 (two peaks)

3500

3000

2500

2000

1500

1000 800

Alcohols and Amines OH or NH CO or CN 3000-3700 cm1 900-1300 cm1

Carbonyls One of the most useful absorbtions in infrared 1640-1820 cm-1 Ketones (saturated) C=O Aldehydes C=O; CH(O) Carboxylic acids C=O; C(O)OH Esters C=O ; C(O)OR 1640-1820 cm1 1640-1820 cm
1

Ethers CO 1050-1260 strong

2820-2900 and 2700-2780 cm1 (weak but characteristic) 1640-1820 cm1 3330-2900 cm1 1640-1820 cm1 1100-1300 cm1

Chapter 13-1
Nuclear Magnetic Resonance (NMR) Spectroscopy Some atomic nuclei (1H, have a nuclear spin.
13

Nuclear Magnetic Resonance Spectroscopy

Chem 61

C, others) behave as if they are spinning...they

Spinning of a charged particle creates a magnetic moment. If an external magnetic field is applied, these small magnetic moments (of the nuclei) either align with the field () or against the field (), about 50% with and 50% against the field at any one time.

Ho
Ho = the external magnetic field

Ho E

Resonance: the flip of the magnetic moment from parallel to antiparallel to the external magnetic field. Irradiation at the frequency equal to the energy difference, E, causes resonance. E depends on the external magnetic field. Protons (or other nuclei) in different magnetic environments resonate at different field strengths. A proton which resonates at a higher field is in a stronger magnetic environment or shielded. A proton which resonates at a lower magnetic field is said to be deshielded. Different magnetic environments are created by different electron densities in the vicinity of a proton.

Chapter 13-2

Nuclear Magnetic Resonance Spectroscopy

Chem 61

Adjacent electron withdrawing groups, highly electronegative atoms, or the hybridization of the carbon to which the proton is bonded can alter the magnetic environment. The local electrons create a small electric and magnetic field around a proton and shield it. The more electron density present around the proton, the greater the field and the greater the shielding. Resonances are reported in chemical shifts () downfield from tetramethylsilane (TMS) (CH3)4Si.

The pi system of benzene creates a magnetic field or ring current which deshields the protons attached to the ring. Similarly, pi electrons in a C=O bond create a field which deshields the proton bonded to the C=O of an aldehyde. This is also affected by the inductive effect of the C=O.

distance from TMS in Hz MHz of spectrum

ppm

In methyl halides, the more electronegative the halogen, the more deshielded the protons on the methyl. This is because F is inductively more electron withdrawing, causing the carbon to be more positive and thus pulling more electrons away from the hydrogen and causing it to be less shielded. In methyl halides, the more electronegative the halogen, the more deshielded the pr

H3CF 4.3

H3CCl 3.0

H3CBr 2.7

H3CI 2.1

Pi electron effects Magnetic fields created by pi electrons are directional and said to have an anisotropic effect.

R Ho H C O H H deshielded H deshielded

Chapter 13-3
Equivalent and Nonequivalent Protons

Nuclear Magnetic Resonance Spectroscopy

Chem 61

Protons that are in the same magnetic environment are equivalent and have the same chemical shifts. Protons in different magnetic fields are nonequivalent and have different chemical shifts. Magnetic equivalence is usually the same as chemical equivalence. Equivalence can be established by symmetry operations such as rotation, mirror planes and centers of symmetry Chemically equivalent protons have the same chemical shifts. To determine if protons are chemically equivalent, replace one by a different group, e.g. D or Br. Then replace a different one by the same group and compare the two compounds. If they are identical, the protons are equivalent. H H C H H C H equivalent, but not to CH3 protons all six are equivalent equivalent Equivalent protons can be on different carbons. Protons which are homotopic or enantiotopic resonate at the same chemical shift in the NMR. If protons are interconverted by rotation about a single bond, they will average out on the NMR time scale and a single resonance will be observed. ClH2CCH2Cl anti and gauche forms rapidly interconvert and a single resonance is observed. Axial and equatorial hydrogens in cyclohexane average to a single peak because of rapid ring inversion. Diastereotopic hydrogens are chemically nonequivalent and thus give different chemical shifts in the NMR OH H H C H H C H C H

Cl H

Chapter 13-4
Intergration

Nuclear Magnetic Resonance Spectroscopy

Chem 61

The spectrometer can integrate and determine the relative number of hydrogens associated with each resonance in the NMR spectrum by determining the area under the peaks. Spin-Spin Coupling for example...

3
CH3CH2OCH3

3 2

TMS If a proton (Ha) is bonded to a carbon which is bonded to a carbon that has one proton (Hb), Ha will appear as a doublet Since in half the molecules, Hb will be in the state and in half will be in the state, Ha will experience two different magnetic fields and two peaks (a doublet) will appear for Ha.

Ha without an adjacent hydrogen For one adjacent hydrogen or Ha with Hb adjacent in the state Ha with Hb adjacent in the state

Chapter 13-5
For two adjacent hydrogens: Hb, Hc

Nuclear Magnetic Resonance Spectroscopy

Chem 61

At any one time Hb or Hc could be in the or state (50:50) thus 4 combinations for Hb, Hc exist: bc bc bc bc gives 1:2:1 triplet

When both Hb and Hc are , a different field is observed than if both are or one is and one is . When one is and one is , the field is the same. That is, bc and bc produce the same field and a single signal for Ha is observed with twice the intensity. Thus three signals are observed in a 1:2:1 ratio: a so-called triplet For three adjacent protons: 1:3:3:1 quartet Thus the splitting pattern of a particular proton or equivalent protons will be a pattern with n+1 lines where n is the number of adjacent equivalent protons. singlet 0 neighboring protons doublet 1 neighboring protons triplet 2 neighboring protons quartet 3 neighboring protons quintet 4 neighboring protons sextet 5 neighboring protons septet 6 neighboring protons The separation of the peaks in a splitting pattern is called the coupling constant, J.

Chapter 13-6
Splitting Diagrams

Nuclear Magnetic Resonance Spectroscopy

Chem 61

Splitting patterns for protons can be constructed in diagram form by starting with one line to represent the unsplit proton resonance. If an adjacent proton Hb affects Ha it is split into a doublet; if another equivalent proton to Hb is present, each line of the double will be split into a doublet, since the coupling constant J is the same, the two center lines overlap and a only three lines are observed with the center line twice the height. This can be repeated for additional adjacent protons.

Ha without an adjacent hydrogen

splitting diagram

Ha split by one adjacent hydrogen

Ha split by a second adjacent hydrogen

Ha split by a third adjacent hydrogen 1 3 3 1

Chemical Exchange and Hydrogen Bonding CH3OH, methanol would be expected to give an NMR spectrum of a doublet for the CH3 and a quartet for the OH. For a dilute sample at -40 in CCl4 this is the case. If the NMR spectrum is run at 25 as a more concentrated sample only two singlets are observed. This is because the intermolecular hydrogen bonding in methanol allows the rapid exchange of the OH proton from one CH3OH molecule to another, effectively averaging the spin states of the OH proton and resulting in no change in the magnetic field due to the OH. Amines and other compounds which can undergo hydrogen bonding can also show this effect. Thus the NMR spectra of alcohols, amines and carboxylic acids are temperature, concentration and solvent dependent.

Chapter 13-7
CHEMICAL SHIFTS Functional Group Shift, Primary alkyl, RCH3 Secondary alkyl, RCH2R Tertiary alkyl, R3CH Allylic, R2C=CCH2R Benzylic, ArCH2R Iodoalkane, RCH2I Bromoalkane, RCH2Br Chloroalkane, RCH2Cl Ether, RCH2OR Alcohol, RCH2OH Ketone, RCH2C(=O)R Aldehyde, RCH(O) Terminal alkene, R2C=CH2 Internal alkene, R2C=CHR Aromatic, ArH Alkyne, RCCH Alcoholic hydroxy, ROH Amine, RNH2 0.8-1.0 1.2-1.4 1.4-1.7 1.6-1.9 2.2-2.5 3.1-3.3 3.4-3.6 3.6-3.8 3.3-3.9 3.3-4.0 2.1-2.6 9.5-9.6 4.6-5.0 5.2-5.7 6.0-9.5 1.7-3.1 0.5-5.0 (variable) 0.5-5.0 (variable)

Nuclear Magnetic Resonance Spectroscopy

Chem 61

CHEMISTRY 61 Exam 1 Dr. M.T. Crimmins September 15, 1998 I. 1. CH 3

Name___________________________________________ Pledge: I have neither given nor received aid on this exam. Signature________________________________________

Nomeclature (12 points) Give the IPUAC name for the following compounds: Indicate R, S, cis, trans, E, or Z where appropriate.

CH 2CH 2CH 2CH 3 CH 2 C H C H CH 2 CH 2 C H CH 3 CH 3 CH 3

______________________________

2. H 3C H C C CH 3

CH 2CH 3 ______________________________

3. CH 3 H 4. CH(CH3)2 ______________________________ II. A. Write valid Lewis structures for the following species. Show all nonbonding (unshared) electrons and indicate any formal charges . (6 points). 5. [H2COH]+ 6. IO4 ______________________________

7. Give the hybridization of the indicated atoms in the species below (6 points) O H 3C C O CH 3

H 3C C N:

H 3C N CH 2 ..

8. Draw three structural isomers for C3H6O. Indicate what type of functional group is represented by each compound (e.g. carboxylic acid) (6 points).

9. Draw all the possible stereoisomers of 3-bromo-2-butanol. Indicate if they are chiral, meso or achiral and indicate their relationship to each other. (i.e. enantiomers, diastereomers) (8 points)

10. Draw both chair conformations of trans-1,2-dimethylcyclohexane. If one is more stable than the other, circle it. (6 points)

11. Draw an energy diagram for one 360 rotation about the C3-C4 CC bond of hexane. Also draw a Newman Projection of the most stable conformation. (8 points).

12. Circle the molecule(s) which have a permanent dipole. In those which have a permanent dipole, show the direction of the overall dipole. (6 points) O H C H Cl Cl B

Cl

H 3C O H

13. In the space to the right, indicate if each of the pairs of molecules below are identical compounds, enantiomers, diastereomers, structural isomers, or conformational isomers. (9 points).

CH 3 H 3C CH 3 Br H b. H OH H 3C

CH 3

a.

CH 3

________________________

Br H H OH ________________________

c.

H CH 3 C C H 3C H

H 3C CH 3 C C H H

________________________

14. What two effects cause cyclobutane and cyclopropane to be higher in energy than cyclohexane? (3 pts)

15. Label the species below as Lewis Acids or Lewis Bases (4 points) Br + _____________ H3CO _____________

16. Circle the following which has the highest heat of combustion per CH2 unit. (4 points) a. cyclopentane b. cyclopropane c. cyclohexane d. cyclobutane 17. Indicate the geometry of carbon in the molecules below (e.g. trigonal bipyramidal). (6 points) a. CH3CH3 b. H2C=O c. HCCH

____________

______________

______________

18. Circle the statement(s) which are true of enantiomers. (4 points)

a. They have a non-superimposable mirror image. b. They have no asymmetric carbon atoms. c. They are chiral. d. They do not rotate the plane of polarized light. 19. Draw an energy diagram of the molecular orbitals of the C=C bond of ethylene (H2C=CH2) and label them (e.g. ) and indicate their relative energies. Indicate the ground state electronic configuration of the C=C electrons. (6 points)

20. What kind of molecular orbital results (, *, , *) results when the pairs of orbitals show below are combined in the indicated manner? (6 pts)

a.

b.

c.

CHEMISTRY 61 Exam 1 Dr. M.T. Crimmins September 21, 1999 I. 1. CH 3CH 2

Name___________________________________________ Pledge: I have neither given nor received aid on this exam. Signature________________________________________

Nomeclature (12 points) Give the IPUAC name for the following compounds: Indicate R, S, cis, trans, E, or Z where appropriate. CH 2CH 2CH 2CH 3 CH 2 C H C H C H CH 2 C H CH 3 CH 3 CH 2CH 3 CH 3

____________________________

2. C(CH3)3 CH 3 _____________________________ 3. H 3C CH 2CH 3 C CH 2CH 2CH 3 H

_____________________________

4. OH _____________________________ 5. A. Write valid Lewis structures for the following species. Show all nonbonding (unshared) electrons and indicate any formal charges . (6 points). CH2N2 CH3

6. Give the hybridization of the indicated atoms in the species below (6 points) CH 3 H 3C N CH 3 ..

H 2C C C(CH3)2

O H 3C C O CH 3

7.

Write structures for the each of the following having a molecular formula of C4H8O (6 points). a. a n aldehyde b. a cyclic alcohol c. an ether

8. What intermolecular forces exist between molecules of each of the following. (6 points). a. CH3CH2CH2CH2CH3 .. CH 3 S CH 3 b. O _________________________

_________________________ _________________________

c. CH3CH2OH

9. Draw all the possible stereoisomers of 3,4-dibromohexane. Indicate if they are chiral, meso or achiral and indicate their relationship to each other. (i.e. enantiomers, diastereomers) (8 points)

10.

Label the following molecules as chiral or achiral. OH HO H H OH CH3 CH2OH H C CH2CH3

CH3 ______________

_____________

______________

11. Draw both chair conformations of trans-1,2-dimethylcyclohexane. If one is more stable than the other, circle it. (6 points)

12. Draw an energy diagram for one 360 rotation about the C2-C3 CC bond of butane. Also draw a Newman Projection of the most stable conformation. (8 points).

13. Circle the molecule(s) which have a permanent dipole. In those which have a permanent dipole, show the direction of the overall dipole. (6 points)

14. In the space to the right, indicate if each of the pairs of molecules below are identical compounds, enantiomers, diastereomers, structural isomers, or conformational isomers. (9 points).

CH 3 H 3C CH 3 H 3C

CH 3 H 3C CH 3

CH 3 H 3C CH 3

CH 3

a.

CH 3

________________________ Br H b. H OH Br H H OH ________________________

c.

H CH 3 C C H 3C H

H 3C CH 3 C C H H

________________________

15. What effect(s) cause cyclobutane and cyclopentane to be non-planar? (3 pts)

16. Label the species below as Lewis Acids or Lewis Bases (4 points) H3C+ _____________ H2C=CH2 _____________

17. Indicate the geometry of carbon in the molecules below (e.g. trigonal bipyramidal). (6 points) CH 3 H 3C C CH 3 H 2C C CH 2

CH 4

____________

______________

______________

18. Draw and label the atomic orbitals which combine to form the molecular orbitals of formaldehyde, H2C=O. . (6 points)

19. What kind of molecular orbital results (, *, , *) results when the pairs of orbitals show below are combined (mathematically) in the indicated manner? (6 pts)

a.

b.

c.

CHEMISTRY 61 Exam 2 Dr. M.T. Crimmins October 22, 1998 I. 1. CH 3

Name___________________________________________ Pledge: I have neither given nor received aid on this exam. Signature________________________________________

REACTIONS: Predict the major organic products of the following reactions. INDICATE STEREOCHEMISTRY AS NEEDED. (4 points each)

HI

2. HBr CH 2 peroxide

3. H C H C CH 2CH 3 2. H2O 2, NaOH H 1. BH3

4. CH 3 H2, Pd/C

CH 3 5. H 3C C C CH 2 + CH 2 H H 3C C C H heat

H 3C 6. H 3C

CO 2 CH 3

CH 3

Cl2 1 equivalent

7. H 2C C C H HBr, -80C CH 2

H 8.

CH 3

1. Hg(OAc)2, H2O 2. NaBH4

II.

Multiple Choice: Place the letter in the blank and Circle the best answer (only one). (4 points each)

___9. The rate limiting step for hydration of an alkene with water and acid is a. b. c. d. protonation of the alkene by a strong acid addition of water to a carbocation to form the protonated alcohol loss of a proton from the protonated alcohol to form the alcohol. simultaneous addition of H+ and HO to the alkene.

___10. Which of the following free radicals is the most stable? a. CH 3 H 3C C c. CH 3 H 3C C c. CH 3 H C d. CH 3 H 3C C

CH 3

___11. Which of the following indicated hydrogens is the most acidic? a. CH3CH=CH2 c. CH3CH2CH3 b. CH3CCH d. HCH2CH2OCH3

___12. In the addition of HBr to 1,3-butadiene the 1,2 product predominates at -80C while the 1,4 product predominates at 40C. The 1,4 product is said to result from

a. b. c. d.

kinetic control. thermodynamic control. a Diels Alder reaction. the s-cis diene.

___13. Which of the following alkenes would have the lowest heat of hydrogenation? a. H 3C C H3C C CH3 H2C CH2 CH3 b. H 3C CH2 CH3 C CH CH3 c. H 3C H C CH2 d. H H 3C H3C H C CH 3 CH CH H3C CH3 CH C CH 2

___14. What is the stereochemical relationship of the products of the following reaction? H C C H Br2

H 3C a. b. c. d. e.

CH 2CH 3

diastereomers enantiomers identical (only one stereosiomer of the product is formed). cis-trans isomers conformational isomers

___15. Which of the carbocations below is the most stable? a. H H C+ CH 3 b. H C+ CH 3 c. + CH 2 C d. CH3 C+ CH 3

___16. What is the hybridization of the positively charged carbon in the carbocation below? + CH 2 C

a. sp3 V. 17. H3 C C C H

b. sp2

c. sp

d. s

Mechanisms. Give a stepwise, detailed mechanism with arrows and intermediates for the following reactions. Be sure to account for stereochemistry as needed. (6 points each) Br2 H3 C H CH3 Br C C CH3 H Br

18. CH3 C CH2 CH3 X+ Y CH3 Y X C CH2 CH3

Syntheses: Give reagents to carry out the transformations below. (5 points each) 19. H C H C H H ClCH 2CH 2Cl

20. O H 3C C C H H 3C C CH 2CH 3

21. Consider the energy diagram below and answer the questions using the letters on the diagram. (10 points). B D C E N E R G Y E M A L G PROGRESS OF REACTION a. What point(s) in the diagram represent transition states? b. What point(s) in the diagram represent intermediates? c. What is the energy of activation for the reaction? d. What is the rate-limiting step in the reaction? e. Is the reaction endergonic or exergonic? f. What is the free energy change of the reaction? g. Does G or C form faster from E? _______________ _______________ _______________ _______________ _______________ _______________ _______________ J H N F K

22. Draw the HOMO (highest occupied molecular obrital) and the LUMO (lowest unoccupied molecular obrital) of 1,3-butadiene and label them. Circle the one which would interact favorably with the ethylene orbital below in a Diels-Alder reaction. (4 points)

CHEMISTRY 61 Exam 2: October 21, 1999 Dr. M.T. Crimmins

Name___________________________________________ Pledge: I have neither given nor received aid on this exam. Signature________________________________________

I.REACTIONS: Predict the major organic products of the following reactions.


INDICATE STEREOCHEMISTRY AS NEEDED. 1. CH3 1. Hg(OAc)2, 2. NaBH4 2. CH3 C C H 3. HCl (1 equiv) CH3CH2 4. CH3 H 5. CH3CH2 C C CH2CH3 H2 poisoned catalyst (Pd/BaSO4) 6. H3 C C CH2 H3 C 7. H C C H 8. H3 C C CH2 H3 C 9. CH3CH2 C C H NaNH2 CH3CH2Br H2O, H+ CH3 HBr, peroxide Br2, H2 O Br2 C C H CH3 CH2CH3 1. BH3 2. H2O2, HO H 2O (4 points each)

II.

Multiple Choice: Circle the best answer (only one). (3 points each)

10. Which of the following alkenes is the most stable? CH3 a. CH3 11. b. CH3 CH3 c. CH3 CH2 d. CH3 CH3

Which of the following is the least stable carbocation? a. H2C=CHCH2+ b. (CH3)3C+ c. C6H5(CH3)2C+ d. CH3CH2+

12.

In the following reaction what is the relationship of the products formed? H CH3 a. enantiomers b. meso compound c. structural isomers d. diastereomers Br2

13.

The carbon -carbon triple bond of an alkyne is composed of_________ a. b. c. d. two bonds and one bond three bonds one bond and two bonds three bonds

14.

The free energy of reaction is a. b. c. d. e. the difference in energy between the reactants and an intermediate in the reaction the difference in energy between the reactants and the transition state the difference in energy between the reactants and the products the difference in energy between the transition state and the products the difference in energy between the intermediate and the products

15.

What is the hybridization of the positively charged carbon in H3C+ a. p b. sp2 c. sp d. sp3 e. d2sp3 f. s

16.

A secondary cation is more stable than a primary carbocation because of a. b. c. d. overlap of a filled p orbital with an adjecent * antibonding orbital overlap of an empty p orbital with adjacent bonding orbitals resonance deduction

17.

Which of the following free radicals is the most stable? a) H3 C C H

CH3

b)

CH3

c)

H3 C

CH3

d)

CH3

CH3

18.

Which of the following is not true of H2C=CH2? a) b) c) d) It contains 5 bonds. It has bond angles of 120. All the atoms are in the same plane. It has free rotation about the C=C bond.

Syntheses: Give reagents to carry out the transformations below. (4 points each) 19. CH 3 C CH2 CH 3 OCH3 CH 3 C CH3 CH3

20. H3 CC CH CH3CH2CH2CH2CH3

21. O HC CCH 2CH3 CH3CH2 C CH3

22.

Draw an energy diagram for the hypothetical exergonic reaction below where B is an unstable intermediate. Label the positions for A, B, and C on the diagram and indicate the energy of activation on the diagram. (5 points). A slow B fast C

23.

Draw resonance structures (4 pts each) for a) benzyl cation

b) acetate ion (CH3CO2)

V. 24.

Mechanisms. Give a stepwise, detailed mechanism with arrows and intermediates for the following reactions. (4 points each) CH3 H3 C C C CH2 H CH3 H2 O, H+ CH3 CH3 C CH CH3 OH CH3

25. H3 C H C C CH3
Br2

H3 C

Br

H3 C

Br H

26. H3 C C H3 C CH2 HBr H3 C CH3 C Br CH3

CHEMISTRY 61 Exam 3 Dr. M.T. Crimmins November 24, 1998 I.

Name___________________________________________ Pledge: I have neither given nor received aid on this exam. Signature________________________________________

Multiple Choice: Place the letter in the blank and Circle the best answer (only one). (4 points each)

___1. The rate limiting step for free radical halogenation is a. b. c. d. initiation hydrogen atom abstraction from carbon by the halogen radical attack of carbon radical on molecular halogen termination

___2. Which of the following are "concerted" reactions? a. SN1 b. SN2 c. E1 d. E2 e. SN1 and E1 f. SN2 and E2

___3. Reaction of a strong base with a tertiary alkyl halide is most likely to result in: a. no reaction b. E2 elimination c. SN1 substitution d. E1 elimination

___4. Which of the following statements is correct?

a. Free radical bromination is more selective than chlorination because the transition state is more reactant-like. a. Free radical bromination is more selective than chlorination because the transition state is more product-like. c. Free radical chlorination is more selective than bromination because the transition state is more product-like. d. Free radical chlorination is more selective than bromination because the transition state is more reactant-like.
___5. Which of the folowing species is the most nucleophilic? a. NH3 ___6. SN1 reactions lead to a. formation of free radicals b. retention of stereochemistry c. racemization d. inversion of stereochemistry ___7. Which of the following would undergo the fastest dehydration reaction in the presence of acid? a. CH 3 b. CH 2 O H c. CH 3 OH OH CH 3 OH d. b. H3P c. H2S d. H2O

___8. Which of the following reactions will proceed the fastest? a. CH 2Cl NaOH b. CH 2 Br NaOH

c. CH 2 Br NaOH

d. CH 3 CH 2 Br NaOH .

II. 9.

REACTIONS: Predict the major organic products of the following reactions. INDICATE STEREOCHEMISTRY AS NEEDED. (4 points each) CH 2CH 3 Br2, light

10. O C H CH 2 CH 3 CH 2 CH 2 OH, H+

11. CH 3 OH 1. NaH 2. CH3I

12. H CH(CH3)2 (CH3)3COK+

Br

13. H O H NaOH, H2 O

C(CH3)3

14. H Br H CH 3 CH3Se Na+

15. H H 3C C C C CH 3 CH 3 O H

CH 3 16.

Br CH 3

1. Mg CH 3CH 2CH 2Br 2. H2C=O 3. H2SO4 V. 17. CH 3 CH 3 OH H3PO4, heat CH 3 Mechanisms. Give a stepwise, detailed mechanism with arrows and intermediates for the following reactions. Be sure to account for stereochemistry as needed. (6 points each)

CH 3

18. CH 4 + Br2 heat or light CH 3 Br + HBr

19. CH 3 CH 3CH 2 O C CH 3 HI excess CH 3CH 2 I + I CH 3 C CH 3

CH 3

CH 3

Syntheses: Give reagents to carry out the transformations below. (5 points each) 20. I

21. O CH 3 O CH 2CH 2CH 3

22.

Only one monochlorination product is obtained from an alkane with the molecular formula C5H12. What is the structure of the alkane? (4 points)

23.

Draw the transition state for the reaction of CH3Br with HO. (4 points)

CHEMISTRY 61 Exam 3 Dr. M.T. Crimmins November 23, 1999

Name___________________________________________ Pledge: I have neither given nor received aid on this exam. Signature________________________________________

II.

Reactions: Predict the major organic product of the following reactions. If more than one product is formed give both and indicate the major product. Indicate stereochemistry where necessary. (4 points each)
H + H CO2CH3 CO2CH3

1.
heat

2.
HBr -80 C

3.
CH3 H3C C CH2 CH2CH3 H Br2, light

4.
H Br NaI H

CH3

5.
H 3C H Br CH3 H 2O low temperature

6.
Br Ph H CH3 C C Ph H (CH3)3CO K
+

7. CH3 Ph C Br CH CH2 CH3OH

8. Indicate if the following compounds are aromatic, non-aromatic or anti-aromatic. (2 points each)
+ H

+ H a. b. c.

d.

e.

9. List three criteria for aromaticity. (6 points) 1.______________________________________ 2.______________________________________ 3.______________________________________ II. Multiple Choice: Place the letter in the blank and Circle the best answer (only one). (3 points each) In the following solvolysis reaction what is the relationship of the products formed?
CH2CH3 C CH2CH2CH3 CH3OH

___10.

H 3C

Br a. enantiomers b. meso compound

c. structural isomers d. diastereomers

___11. Which of the following reactions would proceed the fastest? a. CH3CH2CH3 + Br2 + light CH3CHBrCH3 b. CH3CH2CH3 + F2 + light CH3CHFCH3 c. CH3CH2CH3 + I2 + light CH3CHICH3 d. CH3CH2CH3 + Cl2 + light CH3CHClCH3 ___12. Which of the following is the strongest nucleophile? a. CH3NH b. Cl___13.
a.

c. CH3Od. CH3Se-

Which of the following alkyl halides would undergo the fastest SN2 reaction?
CH2I b. CH2Cl

b. CH3CH2CH2CH2I

d. (CH3CH2)2CHI

___14. Which of the following are "concerted" reactions? a. SN1 b. Diels-Alder reaction c. E1 d. electrophilic addition e. SN1 and E1

___15. Reaction of a hydroxide ion (HO) with a primary alkyl halide is most likely to result in: a. SN2 substitution b. E2 elimination c. SN1 substitution d. E1 elimination

___16. Which of the following statements is correct? a. Free radical bromination is more selective than chlorination because the transition state for bromination is more reactant-like. a. Free radical bromination is more selective than chlorination because the rate limiting step for bromination is more endothermic than for chlorination. c. Free radical bromination is more selective than chlorination because the rate limiting step for bromination is more exothermic than for chlorination. d. Free radical chlorination is more selective than bromination because the transition state for chlorination is more reactant-like. ___17. Which of the following would be the rate limiting step in a free radical halogenation? a. b. c. d. CH3CH2. + Br2 CH3CH2Br + Br. Br2 2 Br. 2 CH3CH2. CH3CH2CH2CH3 CH3CH3 + Br. CH3CH2. + HBr

___18. Ionization to give a carbocation and a leaving group is the rate determining step for a. SN1 b. SN2 c. E1 and E2 d. E1 e. SN1 and SN2 f. SN1 and E1

Syntheses. Give reagents to show how to synthesize the compounds on the right from the compounds on the left. They may require more than one step. (4 pts each) 19.
CH3 CH2OH

20. CH3 CH3 H 3C C CH3 CH CH3 H3 C CH3 CH3 C CH3 C CH2

V.

Mechanisms. Give a stepwise, detailed mechanism with arrows and intermediates for the following reactions. (5 points each)

21.
Br2, light CH 3CH2CH3 CH 3CHBrCH3

22.
Br OCH3 CH3 CH3OH CH3 + CH3

23.
H2C=CH CH=CH2 HBr 40 C H3 CCH CH=CH2 Br + H3 CCH CHCH2 Br

24. The energy diagram for the hypothetical reaction A + B D G + H is shown below. (6pts). C E

F
E

A +B
reaction coordinate

G H

a. What is the rate determining step?_____________ b. What happens to the rate if the concentration of A is doubled? _____________ c. What is the rate expression? rate = _______________

d. If D is a charged species and A and B are neutral, what effect will a polar protic solvent have on the rate of reaction?___________________ e. What is the kinetically favored product?__________ f. What is the thermodynamically favored product?_________

Você também pode gostar